You are on page 1of 72

DEPARTMENT OF PREVENTIVE AND COMMUNITY MEDICINE

Outcomes Recall Analysis Application Total No of


Contents Items
Concept of Health and 3 3
Disease
Natural History of Disease 3 3
Levels of Prevention 1 1 2 4
Epidemiology 18
Definition 1
Descriptive Epidemiology 2 1
Analytic Epidemiology 1
Rates, Ratios and
Proportions 9 2
Disease Outbreak 2
Disease Prevention and 20
Control
General Principles 5
Water Sanitation 4 1
Waste Disposal 1 4
Nutrition 1 4
Population Dynamics 1 1
Research 20
Proposal Writing 3
Data collection 3
Sampling and Sample size 2 1
Research Designs 2
Data analysis 4 1 3
Bias 1
Primary Health Care 4 2 6
Alternative Health Care 3 2 5
Community Organizing 3 1 4
Govt Health Programs 9 2 11
Public Administration 3 2 5
TOTAL 65 28 7 100
University of the East
RAMON MAGSAYSAY MEMORIAL MEDICAL CENTER
College of Medicine
Department of Preventive and Community Medicine
1. The WHO definition of health states that HEALTH is:
A. the state of complete absence of all known disease
B. the harmonious functioning of the human body
C. the state of complete physical, mental and social well-being of the individual
D. feeling of wholeness
2. The three components of ecologic triad that influence occurrence and distribution of diseases are:
A. disease agent, spectrum of disease, environment
B. disease agent, host factors, environmental factors
C. environmental factors, disease agent, spectrum of disease
D. host, disease agent, spectrum of disease
3. This term refers to the ability of the biological agent to invade and enter the host and multiply:
A. infectivity
B. pathogenicity
C. virulence
D. antigenicity
4. The stage in the natural history of disease where interaction between agent, man and environment takes place to determine the actual
occurrence of disease in man, is:
A. prepathogenesis
B. incubation
C. clinical horizon
D. pathogenesis
5. Entry and multiplication of an infectious agent in the body of man or animal before signs and symptoms set in is:
A. pre-pathogenesis stage
B. actual infection
C. incubation period
D. convalescence
6. Pathogenesis stage starts when:
A. disease manifests itself with non-specific signs and symptoms
B. disease manifests itself with symptoms characteristic of the disease
C. once the infectious agent enters the host
D. any of the above
7. Nationwide educational campaign targeting high school students about the ill effects of smoking and alcohol, fall under what level of
prevention?
A. Primary level
B. Secondary level
C. Tertiary level
D. All of the above
8. The objectives of secondary level of prevention include/s:
A. to cure the disease
B. to minimize spread of the disease
C. to reduce serious consequences of the disease
D. All of the above
9. Mass screening for TB in a rural community using Sputum AFB Smear falls under what level of prevention?
A. Primordial
B. Primary
C. Secondary
D. Tertiary
10. Which of the following statements about tertiary level of prevention is correct?
A. it includes activities geared toward disability limitation
B. it prevents social drift among handicapped individuals
C. it includes psychological rehabilitation like crisis intervention
D. all of the above
11. A phenomenon where the proportion of immune individuals in the community has exceeded a significant level such that the infectious organism
is unable to maintain itself, leading to control of further transmission of the disease, is known as:
A. Mass immunization
B. Herd immunity
C. Infection control
D. All of the above
12. The following are characteristics of reservoir of infection, EXCEPT:
A. it is a source or setting where an infectious organism normally lives
B. it provides for the organisms survival and multiplication
C. it permits transfer of infection to a susceptible host
D. None of the above

1
13. Examples of diseases that may be directly transmitted include all of the following, EXCEPT:
A. Hepatitis A
B. Rabies infection
C. Tuberculosis
D. Amoebiasis
14. The practice of close supervision of contracts for the purpose of prompt recognition of the infection without restriction of the their movements, is
called:
A. surveillance
B. isolation
C. segregation
D. quarantine
15. The practice of restriction the freedom of movement of a well person who has recently been exposed to a communicable disease, is called:
A. surveillance
B. isolation
C. segregation
D. quarantine
16. Compared to formula milk, human milk has:
A. More proteins
B. More sugar
C. More vitamin A
D. All are correct
17. The most useful indicator of undernutrition:
A. Skinfold
B. Head circumference
C. Waist circumference
D. Weight for height
18. The following is an indirect method of nutritional assessment:
A. Weight for height
B. Percentage of low birth weight babies
C. Height for age
D. Measles mortality rate
19. The group most vulnerable to nutritional deficiencies:
A. Pregnant women
B. Working group
C. Adolescents
D. Non-lactating mothers
20. A clinical sign seen in kwashiorkor but not in marasmus:
A. Failure to thrive
B. Wasting
C. Edema
D. Attention deficit
21. The most sanitary source of water is:
A. Groundwater
B. Rainwater
C. Surface water
D. Brackish water
22. The primary disinfectant in water treatment:
A. Fluorine
B. Chlorine
C. Aluminum sulfate
D. Ferric sulfate
23. The process of water purification is as follows:
A. Coagulation, sedimentation, filtration, disinfection
B. Coagulation, filtration, disinfection, sedimentation
C. Sedimentation, coagulation, filtration, disinfection
D. Filtration, sedimentation, coagulation, disinfection
24. The material that falls to the bottom of the tank during water treatment is called:
A. Effluent
B. Precipitate
C. Sludge

2
D. Scum
25. Dissolved gases in water are removed by:
A. Filtration
B. Sedimentation
C. Chlorination
D. Aeration
26. The method of waste disposal wherein decomposition occurs through mild reactions mediated by bacteria in the ground over a long period of
time.
A. Incineration
B. Sanitary landfill
C. Dumping
D. Resource recovery
27. The preferred method of human waste management in urban areas:
A. Aqua privy
B. Septic tank
C. Chemical privy
D. Pit privy
28. Unsanitary waste disposal may be associated with the following disease:
A. Malaria
B. Tuberculosis
C. Trichinosis
D. Cholera
29. The qualities of a suitable latrine include:
A. Acceptable to people
B. Simple in construction and use
C. Hygienic and sanitary
D. All are correct
30. The preferred method of hospital waste disposal:
A. Incineration
B. Composting
C. Dumping
D. Landfill
31. Epidemiology is defined as:
A. a study of distribution and determinants of disease in human population
B. a study of the natural history of a disease
C. a study of disease prevention in human population
D. a study of etiologies of epidemic diseases in humans
32. The aim of Descriptive Epidemiology is/are:
A. to establish disease etiology
B. to provide general observations regarding a disease, as to person, place and time
C. to prove cause-and-effect relationship
D. all of the above
33. The component of epidemiology that involves testing of hypothesis of disease causation is known as:
A. Descriptive Epidemiology
B. Analytic Epidemiology
C. Inferential Epidemiology
D. Any of the above
34. Person characteristics that may be used to describe distribution of disease in a population include all, EXCEPT:
A. age
B. sex
C. ethnic group
D. None of the above
35. Which of the following patterns of occurrence of disease DOES NOT exemplify secular changes?
A. decline of stomach CA in the United States in the past 40 yrs
B. decline in deaths due to communicable diseases in the Philippines for the past 20 yrs
C. increased incidence of asthma attacks during cold months of December to February
D. increase in the morbidity rate of Diabetes in the Philippines in the last decade
36. A disease that occurs at an unusual or unexpected number is said to be
A. Epidemic
B. Endemic

3
C. Sporadic
D. Any of the above
37. The first step in outbreak investigation is:
A. establish the diagnosis
B. formulate the hypothesis
C. establish case definition
D. characterize afflicted individuals
38. The rate that measures the probability of a person dying in a year regardless of age, sex, race, etc is:
A. case fatality rate
B. neonatal mortality rate
C. crude death rate
D. specific death rate
39. The age specific death rate:
A. compares population with different age and sex composition
B. is generally higher among males than females
C. is higher in the urban areas than in rural
D. is higher in the extremes of ages
40. Used to describe the relative importance of different fatal disease in the population of different age, sex, occupation etc.
A. specific death rate
B. attack rate
C. cause of death rate
D. proportionate mortality rate
41. The number of death from a given cause in a specified time period divided by the total deaths in the same time peiod is
A. crude death rate
B. indirect death rate
C. case fatality rate
D. proportionate mortality ratio
42. The true mortality trend of a disease declines when there is
A. improved recognition of the disease
B. better prevention of the disease
C. better treatment of the disease
D. B and C
43. The following indices use the midyear population as the denominator, EXCEPT:
A. crude birth rate
B. measles incidence rate
C. crude death rate
D. proportionate mortality rate
44. Infant mortality of a community is determined by the number of deaths of infants less than one year old divided by:
A. the total number of births
B. the number of births
C. the number of babies born alive
D. the midyear population of infants less than 1 year
45. A measure of the risk of death during the first month (28 days) of life is:
A. perinatal mortality rate
B. neonatal mortality rate
C. infant mortality rate
D. fetal mortality rate
46. Post-neonatal mortality has been closely linked to:
A. environmental factor
B. maternal health prior to pregnancy
C. events during pregnancy
D. maternal health during pregnancy
47. A low infant mortality rate suggests:
A. good pre-post-natal systems
B. good infant and maternal nutrition

4
C. good environment sanitation
D. all of the above
48. The rate that measures the numbers of death from causes associated with delivery and diseases of the mother during pregnancy is:
A. infant mortality rate
B. neonatal mortality rate
C. maternal morbidity rate
D. maternal mortality rate
49. A proposed study on whether or not abortion should be legalized is not a good research problem because it does not satisfy this criterion:
A. Intellectual curiosity
B. Researchability
C. Significance
D. Feasibility
50. A researcher comes across several studies which suggest that certain parameters in the chest X-ray are useful in assessing the intravascular
volume of critically ill patients. He decides to validate the study in the Philippines in the hope of identifying a possible alternative to Swan Ganz
catheterization, which is inaccessible and too expensive for most Filipinos. This method of identifying a potential research problem is called:
A. Analysis of needs and practice
B. Serendipity
C. Intellectual curiosity
D. Organized and systematic determination of research needs
51. It is important to formulate a research hypothesis particularly when doing analytic studies because it helps the researcher in:
A. Identifying the most appropriate study design to use
B. Determining the best approach to how subjects are to be selected
C. Anticipating the appropriate statistical analysis to use
D. Estimating the sample size for the study
52. The schema for this study design essentially follows the same process as that of an experimental study:
A. Descriptive
B. Cross sectional
C. Case control
D. Cohort
53. The peculiar feature of a cross sectional study that distinguishes it from other study designs is:
A. Information on both the independent and dependent variables are collected simultaneously at the time of the survey
B. Information on the independent variable is collected first, after which the subjects are followed up later for assessment of the dependent
variable
C. Information on the dependent variable is obtained, after which assessment of the independent variable is made
D. Information on both the independent and dependent variables are obtained by doing either queries or actual observation
54. The effect of an inadequate sample size for a study is:
A. Subjects may not be a random representative of the base population
B. There may not be enough subjects that will develop the outcome, in order to detect a difference
C. The researcher will not be able to apply the appropriate statistical test for the kind of data available
D. The researcher might find an association between independent and dependent variables, even if there actually is none
55. The following method of data analysis is most relevant for the cohort study design:
A. Prevalence odds ratio
B. Prevalence rate
C. Odds ratio
D. Relative risk
56. This is a sampling method wherein every subject falling within a fixed sampling interval is obtained from a population N which is arranged in
some definite way:
A. Simple random sampling
B. Systematic sampling
C. Stratified sampling
D. Cluster sampling
57. In this sampling method, there is proportionate representation of the different classes or subgroups in the population:
A. Simple random sampling
B. Systematic sampling
C. Stratified sampling
D. Multi-stage sampling
58. Methods to increase the accuracy of observation and measurement include the following, EXCEPT:
A. Blinding
B. Training of observers
C. Formulation of operational definition
D. Sample size computation
59. Which method of collecting data is devoid of observer error and bias?
A. Interview
B. Self-administered questionnaire
C. Observation
D. Review of documents
60. This method of collecting data results to accumulation of secondary data:
A. self-administered questionnaire

5
B. observation
C. measurement
D. review of documents
61. The following statements are true regarding bias:
A. it may result both from biologic as well as measurement variations
B. it is a systematic deviation from a true underlying value
C. it may result from sampling error
D. All of the Above
62. Interpret the following: The 95% confidence interval of the relative risk is 2.0 to 4.0.
A. The standard deviation for the relative risk is 1.0.
B. There is a 95% probability that the relative risk is between 2.0 to 4.0.
C. The range of all possible risks for the development of the outcome is 2.0 to 4.0.
D. The variance for the relative risk is 1.0.
63. When a researcher wants to test for the association between two categorical variables, the appropriate statistic to use is:
A. Chi square
B. T test
C. Analysis of variance
D. Correlation coefficient
64. In situations where one extremely large or one extremely small observation occurs within a data set, the best measure of central tendency to
use is the:
A. Mean
B. Median
C. Mode
D. Quartile deviation
65. In a normal distribution, normal limits are delineated by:
A. The upper 5% of the observations
B. The lower 2.5% and the upper 2.5% of the observations
C. The lower 5% of the observations
D. The middle 95% of the observations
66. Statistical results of a study relating an independent and dependent variable show that p<0.05. What does this mean?
A. The study is significant enough for its results to be applied by readers in their practice
B. It is very likely that something other than the independent variable was responsible for the observed association
C. The probability that the observed difference is due to chance is very small
D. The independent variable affects the dependent variable to a clinically important extent
67. In studies dealing with performance of a diagnostic test, this term represents the proportion of diseased individuals with a (+) test result
A. Sensitivity
B. Positive predictive value
C. Likelihood ratio (+)
D. Validity
68. This term is synonymous with the True Negative Rate:
A. Specificity
B. Negative Predictive Value
C. Likelihood ratio (-)
D. Accuracy
69. BCG vaccine is given according to the Expanded Program on Immunization of the Philippines, at what age?
A. 3 months
B. 6 months
C. at birth
D. 9 months
70. The Expanded Program on Immunization is concerned with this particular age group:
A. Infants below 1 yr, school entrance, and pregnant women
B. Infants below 6 mos, pre-schoolers, and pregnant women
C. Neonates, pre-schoolers, and post-partum women
D. Infants of any age, school children, and pregnant women
71. In the EPI, one of the following schedules is not true
A. one dose of measles vaccine
B. two doses of BCG
C. three doses of primary DPT
D. three doses of primary OPV
72. A parameter that will help prioritize control measures on fertility to achieve impact on Family Planning Program:
A. general fertility rate
B. age-specific fertility rate
C. infant mortality rate
D. crude birth rate

6
73. The Family Planning Program consists of the following, EXCEPT:
A. promotion of knowledge on the use of contraceptive devices
B. helping childness couples to bear children
C. encouraging legal abortion
D. providing family planning services
74. During a diarrhea bout, breastfeeding of this infant should be
A. stopped and substitute with a lactose-free formula
B. temporarily discontinued
C. given more often
D. Limited
75. A program in the control of communicable diarrheal diseases by the Department of Health:
A. Hydrotherapy
B. oresol therapy
C. physiotherapy
D. use of intravenous fluids
76. The major activities undertaken in the maternal and child health program does NOT include
A. Immunize pregnant mothers against measles
B. Proper pneumonia prevention
C. Promote prenatal care
D. Monitor growth and health status of infants and children
77. The following are the principles of chemotherapy in Tuberculosis:
A. Drugs should be chosen to which the bacilli are likely to be susceptible.
B. Treatment must be started with one effective drug .
C. Bacteriostatic drugs are preferable to bactericidal drugs.
D. When treatment appears to be failing, add another single drug to
improve the situation.
78. The following is the correct definition of treatment failure in TB:
A. Patients who have been previously diagnosed as tuberculosis, who
have had treatment with improvement or disappearance of symptoms
for one or more months, and now returned with clinical evidence of TB.
B. Patients who have failed to attend for 2 or more months in spite of
every effort to trace them.
C. A new patient who is still sputum smear positive 5 months or more after
starting standard treatment.
D. Patients who have moved to another area.
79. At the very least, sputum culture and sensitivity testing for Mycobacterium tuberculosis should be done whenever possible in the following
situations:
A. Smear (-) patients with a strong clinical possibility of PTB and suggestive chest X rays
B. Smear (+) or (-) patients of multi-drug resistant PTB
C. Smear (+) patients demonstrating the rise fall phenomenon
D. All of the above
80. One of the following is NOT descriptive of a young population
A. population pyramid with a wide base
B. median age of 20-24 years
C. crude birth rate greater than 30/1000
D. dependency ratio of 1:1
81. The Philippine Health picture depicts the countrys poor state of health.
Which of the following is reflective of this?
A. increase in the life expectancy of the Filipinos
B. rising mortality rates from lifestyle related diseases
C. present male:female ratio
D. the rising inflation rate
82. The following are considered impediments to health:
A. poor living conditions
B. inability to make decisions about ones health
C. inadequate education
D. all of the above
83. Based on the Alma Ata Declaration, the following were set by governments
and the WHO:
A. freedom from all diseases
B. socially and economically productive life for the people
C. development of infrastructure for communities

7
D. free medicines for all
84. Elements of Primary Health Care include the following:
A. provision of essential drugs
B. proper nutrition for children and mothers
C. control of endemic diseases
D. all of the above
85. The objectives of Primary Health Care include the following:
A. acceptability, accessibility and affordability of health services
B. self-reliance in health care
C. environmental protection
D. all of the above
86. The expanded roles of a physician in Primary Health Care are:
A. social mobilizer
B. administrator of health programs
C. agents for change
D. all of the above
87. Important features of a Community-Based Health Program:
A. preset program objectives
B. managed solely by the peoples organization
C. people participation in all stages of development
D. availability of modern health equipments for community
88. Characteristics of Community-Oriented Primary Care programs:
A. authoritarian
B. paternalistic
C. democratic
D. liberating
89. Which of the following activities aims to gather information regarding the
communitys health state and determinants?
A. community diagnosis
B. community integration
C. purok mobilization
D. education and training
90. The objectives of CBHP are set or formulated by the following:
A. representatives of the people
B. elected officials of the community
C. consultants and other experts
D. members of the clergy
91. One of the following is not a principal strategy utilized in a CBHP:
A. education and training of local human health resources
B. building peoples organization
C. establishment of relevant health services
D. building one hospital for every community
92. Distinct topic or group of tasks arranged over a definite period of time and
place is known as:
A. task analysis
B. training curriculum
C. lesson plan
D. course programme
93. The following characterizes the curriculum for training CHWs:
A. aims to prepare students for their work in PHC
B. this work/job will determine what will be taught rather than the teachers
own interest
C. it will develop the skills of the CHW so that the community they serve
can achieve a high standard of health
D. all of the above
94. The basic principles of community organizing include the following, EXCEPT
A. mobilization of both internal and external resources
B. people have the capacity to change and to bring about change
C. the solution of problems commonly shared by the poorest sector entails
collective organization, planning and action
D. start from the most complex problem because this needs the greatest
considerations and the most resources
95. The following are true with regards to Community Organizing:
A. It is a tool used in achieving the PHC goals of self-reliance and self
determination.
B. It is a method that can be used by community workers to generate
community participation in health services.
C. It is an approach used to prepare the community to eventually take
over the management of its health programs.
D. all of the above are true

8
96. The art and science of preventing diseases, promoting health and prolonging life through organized community effort, is:
A. Preventive Medicine
B. Community Medicine
C. Public Health
D. Social Medicine
97. The most cost-effective system of health care delivery is:
A. socialized health
B. cumpolsary health insurance
C. voluntary health insurance
D. free enterprise
98. Under the Local Government Code the District Health Office is under the:
A. Municipal/City Mayor
B. Provincial Governor
C. Regional Health Officer
D. Secretary of Health
99. The strongest aspect of production in the Philippine Health System is that of:
A. human health resources
B. biomedical technology
C. systems development
D. materials and products
100. The principal advantage of devolution of health services is:
A. decision making is closer to the people
B. graft and corruption is eliminated
C. primary health care is promoted
D. local government units can borrow money

SOURCES:

Beaglehole, Bonita, Kjellstrom; BASIC EPIDEMIOLOGY STUDENTS TEXT, World Health Organization, July 1990

Clark, D., MacMahon, B: PREVENTIVE AND COMMUNITY MEDICINE, 2ND ed 1981

DOH Health Programs Manual

Gupta, M.; Mahajan, B: TEXTBOOK OF PREVENTIVE AND SOCIAL MEDICINE, 3rd ed 2003

Jekel, Elmore & Katz; EPIDEMIOLOGY BIOSTATISTICS AND PREVENTIVE MEDICINE, 1996

Kuzma, Bohnenblust; BASIC STATISTICS FOR THE HEALTH SCIENCES 4th ed., McGraw-Hill 2001

Last, J; Wallace, R; PUBLIC HEALTH AND PREVENTIVE MEDICINE, 13th ed, 1992

Philhealth Manual

Sanchez, Morelos, Baltazar, Peralta; RESEARCH METHODS IN HEALTH AND MEDICINE, Vol. 1 Planning Research (3rd ed.) Philippine Council
for Health Research and Development 1996

9
Blueprint
Community and Family Medicine
(Preventive Medicine and Public Health)

Topics of Importance

- Principles of:
Preventive Medicine
Public Health
Community Medicine
Family Medicine

- Filipino Family, Its Psychodynamic and Health Issues

- Primary Health Care Concept

- Common Health Problems


Implications Leading Causes of Mortality
To the Leading Causes of Morbidity
Country Leading Causes of Infant Mortality
Leading Causes of Maternal Mortality

- Population Dynamics - Current situation and health implications

- Basic Biostatistics and Epidemiology, including Research Process

- Communicable Disease Control

- Basic Health Services in the Country, including:


Maternal and Child Health
Environmental Sanitation
Health Education
Control of Common Disease (Diarrhea and other
Childhood Diseases)

- Current Health Issues in the Country

- Lifestyle Diseases
Coronary Heart Disease
Malignancies
Accidents
Tobacco Related Problems
Etc.
- Tuberculosis (DOTS)

References:
Public Health and Preventive Medicine, 14th Edition
by Marcy, Rosenau and Last

Textbook in Family Medicine


by Rakel

Family Medicine, Principle and Practice by Taylor

DOH Statistics Annual Report

Philippine Health Care Factbook Vol. 1

WHO Prints on Primary Health Care Concept


DEPARTMENT OF PREVENTIVE AND COMMUNITY MEDICINE

Outcomes Recall Analysis Application Total No of


Contents Items
Concept of Health and 3 3
Disease
Natural History of Disease 3 3
Levels of Prevention 1 1 2 4
Epidemiology 18
Definition 1
Descriptive Epidemiology 2 1
Analytic Epidemiology 1
Rates, Ratios and
Proportions 9 2
Disease Outbreak 2
Disease Prevention and 20
Control
General Principles 5
Water Sanitation 4 1
Waste Disposal 1 4
Nutrition 1 4
Population Dynamics 1 1
Research 20
Proposal Writing 3
Data collection 3
Sampling and Sample size 2 1
Research Designs 2
Data analysis 4 1 3
Bias 1
Primary Health Care 4 2 6
Alternative Health Care 3 2 5
Community Organizing 3 1 4
Govt Health Programs 9 2 11
Public Administration 3 2 5
TOTAL 65 28 7 100
PREVENTIVE MEDICINE AND COMMUNITY HEALTH
1. The purpose is to limit the incidence of disease by controlling causes and risk factors
A. Primordial prevention
B. Primary prevention
C. Secondary prevention
D. Tertiary prevention
Answer: B
The property of a test to identify the proportion of truly ill persons in a population who are identified as ill by a screening test
E. Sensitivity
F. Specificity
G. Positive predictive value
H. Negative predictive value
Answer : A
2. The probability of a persons having the disease when the test is positive
A. Sensitivity
B. Specificity
C. Positive predictive value
D. Negative predictive value
Answer: C
3. The extent to which a test is measuring what it is intended to measure
A. Reliability
B. Validity
C. Sensitivity
D. Specificity
Answer: B
4. A study that measures the number of persons with influenza in a calendar year
A. Cohort study
B. Case control
C. Cross sectional
D. Case report
Answer: C
5. Stage by which the presence of factors favor the occurrence of disease
A. Stage of susceptibility
B. Stage of presymptomatic disease
C. Stage of clinical disease
D. Stage of disability
Answer: A
6. Modes of horizontal transmission of disease, except
A. Contact
B. Vector
C. Common Vehicle
D. Genetic
Answer:D
7. An infected person is less likely to encounter a susceptible person when a large proportion of the members of the group are immune
A. Active immunity
B. Passive immunity
C. Herd immunity
D. Specific immunity
Answer: C
8. Occurrence in the community of a number of cases of disease that is unusually large or unexpected
A. Endemic
B. Epidemic
C. Pandemic
D. Infection
Answer: B
9. Measures of central tendency, except
A. Mean
B. Median
C. Mode
D. Variance
Answer: D
10. Range of values surrounding the estimate which has a specified probability of including the true population values
A. Standard deviation
B. Standard error
C. Confidence interval
D. Correlation coefficient
Answer: C
11. The probability of rejecting the null hypothesis when it is true
A. Type 1 error
B. Type 2 error
C. Power of a statistical test
D. Level of significance
1
Answer: A
12. The following are measures of disease frequency, except
A. Incidence rate
B. Prevalence
C. Cumulative incidence
D. Relative risk
Answer: D
13. The proportion of cases of a specified disease or condition which are fatal within a specified time
A. Morbidity rate
B. Case fatality rate
C. Proportionate mortality
D. Death rate
Answer: B
14. The relation between exposure and disease is considered to be causal or etiological in the following, except
A. Dose response relation
B. Cessation of exposure
C. Temporal relation
D. No confounding
Answer: D
15. A study that measures the incidence of a disease
A. Case report
B. Cross sectional
C. Case control
D. Cohort
Answer:D
16. A study wherein bias is less likely to occur
A. Case report
B. Cross sectional
C. Case control
D. Cohort
Answer: D
17. The proportion of disease incidence that can be attributed to a specific exposure
A. Relative risk
B. Odds ratio
C. Attributable risk
D. Potential risk
Answer: C
18. All of the following are potential benefits of a randomized clinical trial, except
A. The likelihood that the study groups will be comparable is increased
B. Self-selection for a particular treatment is eliminated
C. External validity of the study is increased
D. Assignment of the next subject cannot be predicted
Answer: C
19. Recall is an example of what type of bias
A. Selection bias
B. Information bias
C. Confounding
D. Systematic
Answer: B
20. Type of design where both exposure and disease are determined simultaneously for each subject
A. Case study
B. Cross sectional study
C. Case control study
D. Cohort study
Answer: B
21. A study is conducted to determine the proportion of persons in the population with PTB using AFB sputum for diagnosis
A. Case study
B. Cross sectional study
C. Case control study
D. Cohort study
Answer: B
22. Randomization is the best approach in designing a clinical trial in order to
A. Achieve predictability
B. Achieve unpredictability
C. Achieve blinding
D. Limit confounding
Answer: B
23. Type of sampling whereby subjects are assigned according to a factor that would influence the outcome of a study
A. Simple random sampling
B. Systematic sampling
C. Stratified random sampling
D. Cluster sampling

2
Answer: C
24. The extent to which a specific health care treatment, service, procedure, program, or other intervention produces a beneficial result under ideal
controlled conditions is its
A. Effectiveness
B. Efficacy
C. Efficiency
D. Effect modification
Answer: B
25. What is the desired body weight of a 7 month old infant weighing 3600 gms at birth?
A. 8.8 kgs
B. 9 kgs
C. 6.5 kgs
D. 7.8 ks
Answer: D
26. Infants double their weight at

A. 6-7 mos
B. 9-10 mos
C. 5-6 mos
D. 3-4 mos
Answer: C
27. What is the total energy requirement for a 50 kg housewife without househelp?
A. 2000 cal/day
B. 1800 cal/day
C. 2100 cal/day
D. 2200 cal/day
Answer: C
28. The symptom that appears to consistently differentiate between PTB and non-TB respiratory disease

A. Night sweats
B. Anorexia
C. Chronic cough
D. Hemoptysis
Answer: C
29. Asymptomatic PTB is most prevalent in what group?

A. Infants
B. Smokers
C. Young Children
D. Older age group
Answer: D
30. What should be the first test done when confronted with a patient in whom there is clinical suspicion of PTB?

A. Chest X-ray
B. Sputum AFB
C. Sputum GS/CS
D. All of the above
Answer:
31. What is the ideal timing of the three sputum samples?
A. First specimen after time of consultation at home, Second specimen early morning sputum collected by the patient and Third spot specimen
upon submission of second sputum
B. First specimen at home, Second specimen on the spot at time of consultation and Third spot specimen upon submission of second sputum.
C. First specimen on the spot at time of consultation, Second specimen early morning sputum collected by the patient and Third spot specimen
upon submission of second sputum.
D. First specimen on the spot at time of consultation, Second specimen two days after consultation collected by the patient at home and Third
spot specimen upon submission of second sputum.
Answer: C
32. A Sputum AFB result of (++) means

A. 3-9 bacilli in entire smear


B. 1-9 bacilli /OIF
C. 1-2 bacilli in entire smear
D. 1-9 bacilli /10 OIF
Answer: D
33. What is a significant minimum microscopy result for the presumptive diagnosis of PTB?

A. A report of AFB ++ or more for any one of the submitted sputum specimens
B. If at least one of the 3 specimens is positive
C. A report of AFB +++ or more for any one of the submitted sputum specimens
D. A report of AFB + or more for any one of the submitted sputum specimens
Answer: A

3
34. TB cultures should be done in any of the following situations except
A. Smear (+) patients with fall and rise phenomenon
B. All cases of previously treated for >3months but <6months
C. Smear (+) patients with symptoms highly suggestive of PTB and suggestive C-Xrays
D. All cases of relapse
Answer: C
35. A patient with positive PTB exposure and PPD but lacking in signs of active disease and target organ damage is classified by the American
Thoracic Society as

A. PTB I
B. PTB II
C. PTB II
D. PTB IV
Answer: B
37. In the prevention of Cardiovascular disease, salt restriction should be less than how many grams per day?
A. 2 gms
B. 3gms
C. 4gms
D. 5gms
Answer: D
38. In Cardiovascular disease prevention, one should progressively increase moderate physical activity to
A. 30mins once a week
B. 30 mins twice a week
C. 30 mins three times a week
D. 30 mins daily
Answer: D
39. In patients with history of TIA, heart attack or diabetes, the goal in Blood pressure is

A. < 140 and <90


B. <130 and <80
C. <120 and <70
D. < 110 and < 60
Answer: B
40. Leading cause of Diarrheal disease
A. Enterotoxigenic Escherichia coli
B.Salmonella (non-typhoid)
C.Rotavirus
D. Campylobacter jejuni

Answer: C

41. What degree of dehydration is often seen in a child exhibiting more thirst and sunken eyeballs?

A. No signs
B. Some signs of dehydration
C. Severe signs of dehydration
D. Equivocal
Answer: B
42. In Treatment Plan A, the amount of ORS to be given to a child less than 2 y.o. after each loose stool is

A. 100-200ml
B. 50-100ml
C. 300ml
D. 250ml
Answer: B
43. If the child vomits, you may continue slowly giving small amounts of ORS solution after a minimum of

A. 1 hr
B. 30 mins
C. 2 hrs
D. 10 mins
Answer: D
44. How many grams Sodium Chloride is in the standard World Health Organization ORS formula?

A. 5
B. 20
C. 1.9
D. 3.5
Answer: D

4
45. Which of the following is true regarding appropriate Nutritional Therapy during Diarrhea?

A. decreases stool output


B. shortens duration of illness
C. allows significant weight gain
D. All of the above
Answer: D
46. Judge Reyes wants for all his children and their respective families to live with him and his wife until the time of his death. This is an example of what
structure of family?
A. Nuclear
B. Extended
C. Single Parent
D. Blended
Answer: B
47.Juan and Maria with their five year old son live in their own home and are managing to survive with Juans daily wage. This is an example of what structure
of family

A. Nuclear
B. Extended
C. Single Parent
D. Blended
Answer: A
48. Start of the family life cycle
A. Newly Married Couple
B. Family With Young Children
C. Unattached Young Adult
D. Family With Adolescents
Answer: C
49. Mammography should be done annually in women of what age?

A. 50 y.o. and above


B. 60 y.o. and above
C. 45 y.o. and above
D. 30 y.o. and above
Answer: A
50. Women with prior pregnancy affected by neural tube defects who are planning pregnancy should take this chemo-prophylactic
A. Vit. C
B. Retinoin
C. Vit. B complex
D. Folic Acid
E. Ferrous Sulfate
Answer: D
51. An act penalizing the refusal of hospitals and medical clinics to administer appropriate initial medical treatment and support in emergency or serious cases,
amending for the purpose batas pambansa bilang 702, otherwise known as "an act prohibiting the demand of deposits or advance payments for the confinement
or treatment of patients in hospitals and medical clinics in certain cases". Is also known as:
A Ra 8347
B. Ra 3742
C. Ra 8344
D. Ra 7844
Answer: C
52. An act to advance corneal transplantation in the Philippines, amending for the purpose republic act numbered seven thousand one hundred and seventy
(R.A. no. 7170), otherwise known as the organ donation act . This was enacted on:
A. January 14, 1996
B. December 15, 1997
C. August 14, 1998
D. February 20,1995
Answer: D
53.Republic act. 7875 is also known as the :
a. National organ donation act of 1995
b. National health insurance act of 1995
c. Magna carta for health workers
d. None of the above
Answer: B
54.Republic Act 8423
A. created the PITAHC
B. provided for the Traditional and Alternative Health Development Fund
C. provided for the intensive and continuous scientific study on the herbal plants
D. all of the above
Answer: D

5
55.Consumer act of the Philippines or RA 7394 involves all of the following, except:
A. Protection against hazards to health and safety.
B. Provision of information and education to facilitate sound choice and the proper exercise of rights by the consumer.
C. Involvement of consumer representatives in the formulation of social and economic policies.
D. None of the above
Answer: D
56.The tool of family assessment known as SCREEM evaluates the following aspects of family life
A. Social
B. Cultural
C. Religious
D. All of the above
Answer: D
57.True about a family genogram
A. Graphically represents at least 3 generations
B. Includes even the deceased family members
C. Completely depicts family functioning
D. A and B
Answer: D
58. Family assessment instruments include
A. Family genogram
B. Family APGAR
C. Clinical biography and life events
D. All of the above
Answer: D
59.APGAR family assessment is interpreted by means of
A. Scoring
B. Comparing with a standard table
C. Using a scale of wellness
D. Consultation with a family psychologist
Answer: A
60.The APGAR family assessment measures
A. The newborns color, reflex, and cry
B. Satisfaction of members in the relationships within the family
C. The income of the family
D. The level of communication between husband and wife
Answer: B
61.True about family assessment
A. Should be done for all conscious patients and omitted for comatose patients
B. All the tools of assessment must be used
C. An interview with one member of the family is adequate
D. None of the above
Answer: D
62.Normal family function includes
A. Provide support to each other
B. Create rules that govern the conduct of family
C. Adapting to changes in the environment
D. All of the above
Answer: D
63.The statement that a family is a system means that
A. members have inherent inter-dependence
B. parents take the lead role
C. changes affect each member
D. A and C
Answer: D
64.The family as a unit of care is very special because
A. there is lifelong involvement
B. there are shared genetic and developmental attributes
C. they are confined to one household
D. A and B
Answer: D
65.Members of the nuclear family
A. parents
B. dependent children
C. spouses of the dependent children
D. A and B
Answer: D
66.Data on the family background is not needed when
A. Patient is comatose
B. Patient is caucasian
C. Patient is anti-social
D. None of the above
Answer: D

6
67. Basic areas of family function
A. Biologic
B. Economic
C. Educational
D. All of the above
Answer: D
68.It is the responsibility of the attending physician to explain the following
A. etiology of illness
B. pathophysiology of the illness
C. trajectory of outcome of the illness
D. All of the above
Answer: D
69.The illness trajectory begins with
A. the confirmatory laboratory results
B. the onset of symptoms
C. the onset of therapeutic efforts
D. recovery phase
Answer: B
70.Critical issues in choosing a therapeutic plan
A. financial cost
B. psychological state and preparedness of the patient and family
C. lifestyle and cultural characteristics of a family
D. All of the above
Answer: D
71.True about impact of acute illness in the family
A. emotions are high, and can lead to anger
B. family has no time for psychological adjustment
C. family members become sick in the course of the crisis
D. A and B
Answer: D
72.True about the illness trajectory
A. it is the pathological process of coping with illness
B. knowledge of the trajectory allows the physician to predict and anticipate family response to illness
C. it is the normal course of the psychosocial aspect of disease
D. B and C
Answer: D
73. The following statements are related to the impact of illness
A. Severe illness in parents place children of a family at great psychosocial difficulty
B. Illness sets in motion the processes that are disruptive of family life
C. Illness that is prolonged results in changes in family structure, roles and functions
D. All of the above
Answer: D
74.The five stages of family illness trajectory are the following except
A. Recovery phase
B. Onset of Illness to diagnosis
C. Termination phase
D. Major therapeutic efforts
Answer: C
75. Investigating the illness means
A. exploring the meaning of illness to patient and family
B. obtaining clinical history
C. obtaining laboratory test results
D. All of the above
Answer: A
76. The illness belief model may be influenced by
A. scientific medicine
B. religious beliefs
C. popular account
D. All of the above
Answer: D
77. During the impact phase, the physicians responsibility include
A. Making clinical judgment about the amount of information the patient can absorb
B. Giving support and continuity of care
C. Clarify etiology of illness to address any feeling of guilt among family members
D. All of the above
Answer: D
78. Philhealth is

B. is a government collecting agency


C. is a government agency mandated by law to implement the NHIP
D. All of the above
Answer: C

7
79. Universal coverage means that PhilHealth aims
to give medical coverage
A. to all Filipino citizens
B. to all Filipino citizens and foreign nationals
C. to natural born filipinos only
D. all of the above
Answer: A
80. Participation of health care providers in the NHIP
A. is compulsory
B. is a privilege
C. both A & B
D. neither A nor B
Answer: B
81. Surgical procedures covered by philhealth
A.laparoscopic procedures
B.caesarian section
C.excision biopsy
D.All of the above
Answer: D
82. Current outpatient benefits include
A. chemotherapy, hemodialysis, minor surgical procedures
B. chemotherapy, laboratory services,radiotherapy
C. hemodialysis, dental extraction, cataract extraction
D. all of the above
Answer: A
83. This law refers to the compulsory health
insurance of the government
A. Medicare
B. Republic Act 7872
C. National Health Insurance Act of 1995
D. None of the above
Answer: C
84. NHIP ensures that health services are
A. affordable,acceptable,available,accessible
B. affordable,adjustable,admirable,negotiable
C. affordable and world class
D. none of the above
Answer: A
85 PhilHealth sets standards, guidelines and
procedures prior to allowing doctors and hospitals
to become part of the NHIP. This process is called
A. evaluation
B. accreditation
C. recognition
D. acceleration
Answer: B
86. PhilHealth gives medical coverage to dependents of
members. Dependents are the members
A. parents over 60, children below 21
B. parents over 65, children below 21
C. parents over 60, children below 18
D. parents over 65, children of any age
Answer: A
87. Health care provider in the definition of NHIP
can be any of the following
A. doctor, under-board nurse, midwife
B. hospital, CBHO, HMO
C. doctor, hospital, faith healer
D. surgeon, anesthesiologist, medical intern
Answer: B
88. Non-paying philhealth member refers to
A. a member who has reached age 60
B. retiree who has paid 120 months of continuous
contributions
C. delinquent members
D. A&B
Answer: D
89. Surgical procedures NOT covered by philhealth
A. laparoscopic procedures
B. caesarian section
C. silicone implantation, breast
D. excision biopsy
Answer: C
8
90. In the current EPI, a fully immunized child has
A. 1 BCG, 3 DPT, 3 OPV,1 MMR, 3 Hep B vaccine before 12 months of age
B. 2 BCG, 1 Measles vaccine, 3 DPT, 3 OPV before 12 months of age
C. 1 BCG, 3 DPT, 3 OPV, 1 Measles before 1 year of age
D. 1 BCG, 3 DPT, 3 OPV, 1 Measles, 3 Hep B vaccine before 1 year of age
91. The following are the target population of the EPI
A. all poor children at high risk areas
B. children 1-5 years old in measles high risk areas
C. all children without vaccination
D. infants 0 12 mos old, born to un-immunized mother
92. EPI was successful because it utilized different
implementation strategies which include
A. monitoring, surveillance, health education
B. fee collection, social marketing
C. social mobilization, routine immunization day
D. A&C
93. Outbreak response means that
A. giving polio vaccine to children who have not
completed their OPV once a case of polio is reported
B. giving OPV to all children under 5 regardless
of immunization status once a case of polio is reported
C. giving measles immunization to contacts of a
child with measles regardless of immunization status
D.giving all the EPI vaccinations to a child once
a case of polio is reported in the area
94. In all levels of the government organization,
there are four distinct personnel in the EPI,
they are:
A. immunization officer, health educator
B. cold chain manager, disease surveillance officer
C. marketing officer, purchasing officer
D. A&B
95. It is given to women of child-bearing age
A. Tetanus immunoglobulin
B. Tetanus toxoid
C. Measles vaccine
D. BCG booster
96. When the child has the following condition,
immunization is absolutely contraindicated
A. HIV infection
B. Adverse reaction to the previous immunization
C. A&B
D. None of the above
97.The goal(s) of the EPI is/are
A. Government revenue
B. Healthier workforce
C. Disease prevention
D. B&C
98. You may postpone immunization when the following
conditions are present
A. when the child is hospitalized
B. when the child is febrile
C. when the child is taking antibiotics
D. A&B
99. You may proceed with immunization even when
A. the child has low grade fever
B. the child is preterm
C. then child is malnourished
D. All of the above
100. The following are examples of active immunization
A. BCG & OPV
B. BCG & DPT
C. BCG & Hepa B
D. All of the above

9
10
University of the East
RAMON MAGSAYSAY MEMORIAL MEDICAL CENTER

COLLEGE OF MEDICINE

Department of Preventive and Community Medicine

Contents/outcomes Recall Analysis Application Total


Concepts of health and 3 3
disease
Natural History of Disease 3 3
Levels of prevention 1 1 2 4
Epidemiology 18
General concepts 1
Descriptive Epidemiology 2 1
Analytic Epidemiology 1
Data Measurements, rates, 9 2
ratios and proportions
Disease outbreak 2
Disease prevention and 20
control
General Principles 5
Water sanitation 4 1
Waste disposal 1 4
Nutrition 1 4

Population Dynamics 1 1
Research 20
Proposal writing 3
Data collection 3
Sampling and sample size 2 1
Research designs 2
Data analysis 4 1 3
Bias 1
Primary Health Care 4 2 6
Alternative Health Care 3 2 5
Community Organizing 3 1 4
Govt Health Programs 9 2 11
Public Administration 4 1 5
Total 65 28 7 100
CEBU DOCTORS UNIVERSITY COLLEGE OF MEDICINE
PREVENTIVE AND SOCIAL MEDICINE
1. The term applied to an epidemic occurring within more than one country or territory:
A. Epidemic
B. Endemic
C. Pandemic
D. Sporadic

2. The causative agent for Avian flu is:


A. Influenza virus A
B. Influenza virus B
C. Coronavirus
D. Retrovirus

3. One of the following statements does not belong among the criteria for a SARS
suspect:
A. Moderate to high-grade fever
B. Respiratory symptoms: cough, shortness of breath, or difficulty of breathing
C. History of contact with a person diagnosed with SARS, during the past 4
months
D. Travel to a country with known cases of SARS

4. The trees planted around the landfill help improve the quality of environmental air by
making use of the emitted:
A. Methane
B. Carbon dioxide
C. Carbon monoxide
D. Oxygen

5. Methane is the product of the decomposition of which waste products?


A. Gaseous waste
B. Solid waste
C. Metal waste
D. Chemical waste

6. A Sanitary landfill may receive the following wastes, except:


A. Untreated hazardous wastes
B. Cardboard
C. Grass cuttings
D. Paper

7. Without the plastic lining at the bottom of the landfill, the leachate could:
A. Produce combustible gases
B. Seep through the soil and contaminate the water supply
C. Emit foul odor
D. Make the landfill sink deeper

8. The Zero Waste Management program is primarily intended to :


A. Extend the life of the sanitary landfill
B. Recycle metal and plastic material
C. Give extra income to people
D. Recycle paper into cardboard

9. The ideal way to dispose of hospital waste is by :


A. Burying in a landfill
B. Incineration
C. Grinding
D. Recycling
CEBU DOCTORS UNIVERSITY COLLEGE OF MEDICINE
PREVENTIVE AND SOCIAL MEDICINE
10. Passive immunity may be acquired by the following means, EXCEPT:
A. Intramuscular injection of immunoglobulin
B. Transfer of maternal antibodies across the placental barrier
C. Ingestion of colostum by the baby
D. Injection with viral antigen

11. Oral administration of vaccine is an effective route for which of the diseases?
A. Tetanus
B. Tuberculosis
C. Typhoid fever
D. Hemophilus influenza

Numbers 12 to 14 refer to Case 1:


Case 1 The patients who gave birth at the barangay health center were handled by the
midwife. Some developed complications.
Prenatal No Prenatal
Age Group Number Number with Attack Number Number with Attack
complication Rate complication Rate
15 25 yrs. 33 15 45% 37 20 54%
26 36 yrs. 27 6 22% 31 11 35%
37 47 yrs 15 2 13% 12 11 92%

12. Case 1 : The age group/s with the highest total attack rate of complication is / are:
A. 15 25 yrs. C. 37 47 yrs.
B. 26 36 yrs. D. All of the above

13. Case 1 : The number of patients in the 15 25 yrs old group who had prenatal check-
up is :
A. 15 C. 45
B. 33 D. 48

14. Case 1 : The total number of patients without prenatal check-up who developed
complication is:
A. 20 C. 37
B. 32 D. 42

15. Which of the following is not among the top ten leading causes of morbidity in the
Philippines?
A. Respiratory diseases
B. Diarrhea
C. Influenza
D. AIDS

16. A health care system that has the greatest impact on the health of a society, while
making the best use of its resources, fulfills which of the following values of social
accountability?
A. Relevance
B. Quality
C. Cost-effectiveness
D. Equity

17. Availability of high-quality health care, to all sectors of society, especially the
underprivileged, is the goal of which social accountability value?
A. Relevance
B. Quality
C. Cost-effectiveness
D. Equity
CEBU DOCTORS UNIVERSITY COLLEGE OF MEDICINE
PREVENTIVE AND SOCIAL MEDICINE

18. The WHOs Philippine counterpart in making health policies to answer local needs is
the:
A. Association of Philippine Medical Colleges
B. Department of Health
C. City Health Office
D. Philippine Medical Association

19. The City health officer sent the sanitary inspector with medicine to treat the people of
the town infested with scabies. The action taken is a form of which level of
prevention?
A. Primary prevention
B. Secondary prevention
C. Tertiary prevention
D. Risk assessment

20. Teaching uninfected households about sanitation and hygiene is what level of
prevention?
A. Primary prevention
B. Secondary prevention
C. Tertiary prevention
D. Risk assessment

21. A student who wants to get health information of a given community may obtain it
from the following, except:
A. Local government hospital
B. City health office
C. Provincial health office
D. Department of health

22. Epidemiology differs from clinical medicine in these regards, EXCEPT:


A. Epidemiologists study groups of people, not individuals
B. Epidemiologists study well people, in addition to sick people
C. Epidemiologists try to find out the crucial difference between those stricken
and those spared
D. Epidemiologists decide optimal patient management
CEBU DOCTORS UNIVERSITY COLLEGE OF MEDICINE
PREVENTIVE AND SOCIAL MEDICINE
23. In the investigation of an epidemic of food poisoning at a banquet, high attack rates
were found for people who ate roast beef as well as those who ate mushroom sauce.
Table 1 shows combinations of the two foods that were then considered.

Table 1 Attack Rates for Food Combinations

Ate Mushroom Sauce Did Not Eat Mushroom Sauce

Number Attack Number Attack


Number III Rate Number III Rate
(%) (%)

Ate roast beef 150 105 70 72 2 3

Did not eat roast 42 33 78 26 0 0


beef

Thus, the infective item is most likely to be


A. Mushroom sauce
B. Roast beef
C. The combination of roast beef with mushroom sauce
D. Cooking utensil (not the meat, nor the mushroom sauce)

24. What measures the proportion of the population dying every year or the number of
deaths in the community per 1,000 populations.
A. Case fatality rate
B. Cause-specific mortality rate
C. Crude mortality rate
D. Proportionate mortality

25. Designed to measure the rate at which people without a disease develop the disease
during a specific period of time, that is, the number of new cases of a disease in a
population over a period of time.
A. Fertility rates
B. Incidence rates
C. Mortality rates
D. Prevalence rates

26. Table 3 shows the sex distribution in three large series of cases of a disease.

Table 3 Sex Distribution in Three Series of Cases of a Disease


Series Male Cases Female Cases
1 200 100
2 250 50
3 450 150
Total 900 300

The incidence rate of this disease by sex was


A. Twice as great in males as in females
B. Three times greater in males than in females
C. Five times greater in make than in females
D. Cannot be computed from the date given
CEBU DOCTORS UNIVERSITY COLLEGE OF MEDICINE
PREVENTIVE AND SOCIAL MEDICINE
27. Which of the following measure the number of people in a population who have the
disease at a given point in time.
A. Fertility rates
B. Incidence rates
C. Mortality rates
D. Prevalence rates

28. Epidemic refers to


A. A disease that has a low rate of occurrence but that is constantly present
in a community or region
B. An attack rate in excess of 10 per 1,000 population
C. The occurrence of illnesses of similar nature clearly in excess of the normal
expectation for that population at that time
D. The annual case rate per 100,000 population

29. Case fatality rate for a given disease refers to


A. The crude mortality rate per 100,000 population
B. Cause-specific mortality rate due to the disease
C. The percentage of deaths among cases of the disease
D. The proportion of deaths due to the disease among all deaths from all causes

30. The two major measures of disease frequency are:


A. Sensitivity and specificity
B. Positive and negative predictive values
C. Incidence and prevalence
D. Relative and Absolute risks

31. The ratio of the incidence of the group with the factor to the incidence of the group
without the factor
A. Absolute risk
B. Attributable risk
C. Relative risk
D. Predictive value

32. Measures the amount of the incidence that can be attributed to one particular factor.
A. Absolute risk
B. Attributable risk
C. Relative risk
D. Predictive value

33. Sometimes called as retrospective study because of its direction of inquiry.


A. Case-Control Study
B. Cross-Sectional Study
C. Cohort Study
D. Experiment

34. Because the events of interest transpire after the study is begun, this study is
sometimes called prospective study.
A. Case-Control Study
B. Cross-Sectional Study
C. Cohort Study
D. Experiment
CEBU DOCTORS UNIVERSITY COLLEGE OF MEDICINE
PREVENTIVE AND SOCIAL MEDICINE
35. Because it focuses on a point in time, it is sometimes also called prevalence study.
A. Case-Control Study
B. Cross-Sectional Study
C. Cohort Study
D. Experiment
36. A group of people who share a common experience within a defined time period.
A. Bias
B. Cohort
C. Placebo
D. Stratum

37. Which of the following is not a characteristic of a case-control study?


A. It is relatively inexpensive
B. Relative risk may be estimated from the results
C. Incidence rates may be computed
D. One selects controls without the disease

38. The strongest weapon in the scientific armamentarium to test a hypothesis.


A. Case-Control Study
B. Cross-Sectional Study
C. Cohort Study
D. Experiment

39. A well-known phenomenon in medicine is that patients given only inert substances
will often show subsequent clinical improvement when compared with similar
patients not so treated
A. Block randomization
B. Placebo effect
C. Stratification
D. Surveillance

40. Defined as the detection of the occurrence of health-related events or exposures in a


target population.
A. Predictive Value
B. Sensitivity
C. Specificity
D. Surveillance

41. The __________ of a test is defined as the percentage of persons with the disease of
interest who have positive test results.
A. Negative Predictive Value
B. Positive Predictive Value
C. Sensitivity
D. Specificity

42. The __________ provides a way to look at risk in case-control studies.


A. Absolute Risk Reduction
B. Odds Ratio
C. Relative Risk
D. Relative Risk Reduction

43. The four oclock habit of the Dengue Prevention campaign is about:
A. Immunization
B. Environmental sanitation
C. Personal hygiene
D. Anti-viral treatment
CEBU DOCTORS UNIVERSITY COLLEGE OF MEDICINE
PREVENTIVE AND SOCIAL MEDICINE
44. When educating the patient about Vitamin A, you will not include which of the
following?
A. Yellow squash and carrots are rich food sources
B. Prevents night blindness
C. It is an anti-oxidant
D. Minimum daily requirement is 400 to 600 gms

45. The pathology in Vitamin A deficiency is:


A. Loss of muscle mass
B. Skin hyperpigmentation
C. Xerophthalmia
D. Exophthalmus

46. Inclusion of Iodized salt in the daily diet prevents:


A. Mental deficiency in children
B. Thin, brittle bones
C. Loose, non-elastic skin
D. Attention deficit hyperactivity disorder

47. Long-term psychiatric monitoring of a former drug user, who has been rehabilitated is
a form of:
A. Primary prevention
B. Secondary prevention
C. Tertiary prevention
D. Quartiary prevention

48. The incubation period of AIDS is from the time:


A. Patient is infected up to the time AIDSrelated symptoms manifest
B. The patient starts having large lymph nodes to the time severe lung infection
manifests
C. Patient manifests mild symptoms until he/she dies
D. Patient starts feeling weak until pneumonia develops

49. The SARS virus is transmitted via :


A. Inhalation of infected droplets
B. Drinking contaminated water
C. Eating cooked infected chicken
D. Contact with infected blood

50. The characteristic clinical presentation of a patient with tuberculosis is a combination


of which of the following:
A. Fever throughout the day, nosebleeding (epistaxis), loss of appetite
B. Cough of 2 weeks, evening fever, weight loss, loss of appetite
C. Fever of one week, abdominal enlargement, loss of appetite
D. Weight loss, large neck lymph nodes, anemia

51. Malfunctioning liver, distended abdomen and jaundice are symptoms consistent with:
A. Chronic alcoholism
B. AIDS
C. Protein deficiency
D. Carbohydrate deficiency
CEBU DOCTORS UNIVERSITY COLLEGE OF MEDICINE
PREVENTIVE AND SOCIAL MEDICINE
52. When taking the history of a woman whom you suspect to have STD, you would like
to assess the possibility of exposure (risk assessment) by asking about:
A. Number of children
B. When the symptoms started
C. Number of sexual partners
D. Health of parents

53. The incidence of tuberculosis is correctly stated as (using an arbitrary number):


A. 100,000 new cases per year
B. 100,000 tuberculous persons
C. 100,000 cases of newly diagnosed, plus undergoing treatment and healed TB
cases
D. 100,000 cases confined in hospital

54. Prevalence of tuberculosis is correctly stated as (using an arbitrary number):


A. 2 billion TB cases in the world
B. 2 billion new TB cases in the world per year
C. 2 billion combined treated and healed TB cases per month
D. 2 billion TB cases, excluding the newly diagnosed cases

55. The capacity of a test to become negative in the absence of the disease is:
A. Specificity
B. Sensitivity
C. Predictive value of a negative test
D. Probability

56. The capacity of the test to become positive in the presence of the disease is its:
A. Specifity
B. Sensitivity
C. Predictive value
D. Probability

Numbers 57. to 60 refer to Case 3. Factory employees were screened for tuberculosis
using the PPD tuberculin test and Chest x-ray.
Chest x-ray (+) Chest x-ray (-) Totals
Test (PPD) + A B A+B
Test (PPD) - C D C+D
Totals A+C B+D A+ B + C + D

57. Case 3: The group of subjects/employees with the highest probability of being
tuberculous :
A. A + C
B. A + B
C. A
D. B

58. Case 3: Total number of subjects/employees who are probably tuberculous:


A. A + C
B. B + D
C. A + B
D. C + D

59. Case 3: Predictive value of a positive chest xray:


A. A / A + B
B. B / A + B
C. A / A + C
D. B / B + D
CEBU DOCTORS UNIVERSITY COLLEGE OF MEDICINE
PREVENTIVE AND SOCIAL MEDICINE
60. Case 3: Predictive value of a negative chest xray:
A. A / A + B
B. B / A + B
C. A / A + C
D. B / B + D

61. A reduction in infant mortality may be interpreted as:


A. Increased life expectancy of infants
B. Increased life expectancy of infants and mothers
C. Decreased number of pregnancies
D. Effective family planning programs

62. Morbidity data would include the following, EXCEPT:


A. Accidents in the workplace
B. Deaths from accidents
C. Absences from work
D. Work-related illnesses

63. The best way to reduce the prevalence of heart diseases is to :


A. Train more cardiologists
B. Opening more coronary care units at the barangay level
C. Providing affordable medicines for cardiac diseases
D. Develop intensive health education programs to inform the communities about
prevention

64. DOTS is Department of Health program for:


A. TB diagnosis
B. TB treatment
C. TB prevention
D. A and B only

65. Direct Observation Treatment , Short-course for tuberculosis is focused on which


level of prevention?
A. Primary
B. Secondary
C. Tertiary
D. A and B only

66. The Alma Ata Declaration is about which of the following?


A. Protection of women and children from violence
B. Health as a universal human right
C. Proper garbage disposal
D. Reduction of infant and maternal mortality

67. The science and art responsible for the maintenance and improvement of the health
of the population, with police power to impose sanctions on anyone that may harm
the health of the public is:
A. Preventive medicine
B. Public health
C. General practice
D. Social medicine
CEBU DOCTORS UNIVERSITY COLLEGE OF MEDICINE
PREVENTIVE AND SOCIAL MEDICINE
68. If a clinician practicing in CDH suspects that a deep well is the source of typhoid
fever of three of his patients, the person with the power to have the water source shut
down is:
A. Attending physician himself
B. Sanitary inspector
C. Cebu Doctors Hospital administrator
D. City health officer

69. The opposite of Equality between husband and wife is Paternalism, which means:
A. Dominance of men through their control of the family, commerce and society
B. Men are the superior members of the family through their active participation
in household chores and upbringing of children
C. Men control external affairs, like business, while women control the home
D. Inheritance of wealth is from father to son only

70. The practice of financially-independent adult sons and daughters, who take time to go
home during holidays demonstrates 3 of the following characteristics, except:
A. Shared attributes
B. Belongingness
C. Lifelong membership
D. Child-centeredness

71. When a child kisses the hand of elders as a form of greeting, it is said that the parents
have taught him/her well, fulfilling the following family function, EXCEPT:
A. Biological
B. Socialization
C. Educational
D. Cultural

72. When a child is quarrelsome in school, does not mix well with his classmates, and
does not answer when greeted by classmates, the parents must have performed
poorly in their _______________ function.:
A. Biological
B. Socialization
C. Educational
D. Cultural

Numbers 73 to 75 refer to Case 4: Ms. Ai-Ai has 3 children, aged 8 yrs. old, 5 yrs. 1
yrs., by three different men , but she has remained single. They live with Ai-ais aunt who
has 2 grown up children, aged 29 and 30 years old. Ai-ai says that her children and her
aunt and cousins are the only family for her. What type family does Ai-ai have?

73. Case 4: What type of family does Ai-ai have?


A. Single parent, Nuclear family
B. Single parent, Extended family
C. Blended family
D. Polygamous family

74. Case 4: At what stage of family development is Ai-ais family?


A. Family with newborn child
B. Family with school-age children
C. Family with adolescent children
D. Launching family
CEBU DOCTORS UNIVERSITY COLLEGE OF MEDICINE
PREVENTIVE AND SOCIAL MEDICINE
75. Case 4: At what stage of development is the Aunties family?
A. Family with school-age children
B. Family with adolescent children
C. Launching family
D. Family in later years (Empty nest)

76. At which stage of family development do the parents begin to feel they are no longer
needed by their children?
A. Family with a new born child
B. Family with a child of school-age
C. Family with an adolescent child
D. Launching family

77. When the husband prevents the wife from going out with friends and her own
parents, does not allow her to use the phone, and he shouts and smashes her things
when she disobeys him on such matters; Which of the following forms of controlling
does not apply?
A. Isolation
B. Physical punishment
C. Intimidation
D. Emotional abuse

78. About 60 years ago, only women with higher IQ and grades, with extraordinary talent
than the male competitors, were accepted in medical schools. This is an example of :
A. Male-female power disparity
B. Socialization and learned behavior
C. Medicalization of the problem
D. Trivialization

79. The physician who is not aware of the issues on gender sensitivity, shows concern
only for the cuts and bruises, and does not address the emotional trauma and the risk
to the patients life; such an attitude is called:
A. Male-female power disparity
B. Socialization and learned behavior
C. Medicalization of the problem
D. Trivialization

80. The woman who allows herself to be verbally abused by her husband must be
educated about husband and wife relationship that is based on:
A. Equality and Responsible parenting
B. Equality and Respect
C. Equality and Honesty
D. Equality and Economic partnership

81. The SARS virus may be acquired by:


A. Droplet infection
B. Airborne transmission
C. Sexual contact
D. Fecal-oral transmission

82. The carcinogenic substance in tobacco smoke is:


A. Carbon monoxide
B. Cyanide
C. Nicotine
D. Tar
CEBU DOCTORS UNIVERSITY COLLEGE OF MEDICINE
PREVENTIVE AND SOCIAL MEDICINE
83. Which of the following is a primary prevention activity for drug addiction?
A. Imprisonment of drug pushers
B. Responsible parenting seminar
C. Submission of a drug addict for rehabilitation
D. Drugs test for applicants of drivers licence

84. Which of the following diseases cannot be prevented by hygienic methods of


handling food and drinks?
A. Hepatitis A
B. Hepatitis B
C. Amebiasis
D. Typhoid fever

85. Which of the following conditions cannot be classified as a case of malnutrition?


A. Obesity
B. Kwashiorkor
C. Marasmus
D. Diabetes mellitus

86. Which of the following is/are solely the responsibility/ies of the public health
practitioner?
A. Prompt diagnosis and treatment
B. Report the case to proper authorities
C. Trace and identify other sexual contacts of the index case
D. A and B only

87. The most common disease brought home by a child is:


A. Intestinal parasitism
B. Dengue fever
C. Respiratory tract infection
D. Measles

88. The appropriate term to describe Binos situation, who continues to smoke 2 packs of
cigarettes daily inspite of the chronic cough is:
A. Tolerance to tobacco
B. Tobacco dependence
C. Addiction to tobacco
D. Misuse of tobacco

89. Bino has progressively increased the dose of diazepam, to put him to sleep, from 10
mg to 30 mg daily over 2 years, which is due to:
A. Tolerance
B. Resistance
C. Withdrawal
D. Misuse

90. Nurses and physicians must protect themselves from acquiring HIV infection by:
A. Getting vaccinated
B. Wearing a mask
C. Not touching the patient
D. Following precautionary measures in handling used hypodermic needles
CEBU DOCTORS UNIVERSITY COLLEGE OF MEDICINE
PREVENTIVE AND SOCIAL MEDICINE
91. The most powerful risk factor predisposing persons to atherosclerotic peripheral
occlusive disease is:
A. Alcohol abuse
B. Cigarette smoking
C. Old age
D. Male gender

92. Being a good role-model for the children is about:


A. Economic partnership
B. Responsible parenting
C. Honesty and accountability
D. Trust and support

93. Negotiation and fairness means:


A. Supporting her goals in life
B. Economic partnership
C. Earning trust and confidence through honesty
D. Seeking mutually satisfying resolution to conflict

94. Talking in loud, angry voices, when a couple is trying to find a solution to conflicting
opinions is the opposite of :
A. Shared responsibility
B. Responsible parenting
C. Non-threatening behavior
D. Honesty and accountability

95. Which of the following functions is fulfilled when the children stay healthy because
of completed immunizations?
A. Biological
B. Educational
C. Socialization
D. Cultural

96. Teaching the child to steal is a distortion of which function?


A. Biological
B. Educational
C. Socialization
D. Cultural

97. The type of abuse when the spouse does not speak to his partner, without any
explanation, but only to express anger or dissatisfaction.
A. Physical
B. Sexual
C. Psychological
D. Social

98. The type of abuse, when the wife is not allowed to meet friends and family members;
not allowed to go out of the house.
A. Economic
B. Psychological
C. Social
D. Verbal
CEBU DOCTORS UNIVERSITY COLLEGE OF MEDICINE
PREVENTIVE AND SOCIAL MEDICINE
99. Blaming the rape victim for the assault, because she wore make-up and a mini-skirt
(victim-blaming) is a promotive factor in the causation of violence, which is known
as:
A. Sex-role sterotype
B. Medicalization
C. Socialization
D. Trivialization

100. When the wife refuses to have sex with her husband and he forces her, he can be
charged in court for:
A. Physical injuries
B. Marital rape
C. Battering
D. Frustrated homicide
DAVAO MEDICAL SCHOOL FOUNDATION
College of Medicine
PREVENTIVE MEDICINE
.
1. The main thrust in the control of diarrhea cases by the Department of Health is:
A. Environmental Sanitation
B. Oral Rehydration Therapy with ORESOL
C. Public education
D. Immunization
2. Which of the following child survival strategies complement Oral Rehydration by decreasing the incidence and severity of diarrheal infection?
A. Growth Monitoring
B. Immunization
C. Breastfeeding
D. Mothers education on child care
3. The probability that a positive result is truly indicative of the disease or condition is:
A. None of these
B. Sensitivity
C. Specificity
D. positive
4. Public health services must give emphasis on:
A. Prevention, promotion and maintenance of health
B. Treatment of diseases
C. Rehabilitation of the differently abled individuals
D. Provision of emergency and first aid services
5. In public health, the most frequent problem is scarcity of resources. This can be best met by:
A. Transfer of funds
B. Compete for national funds
C. Allocate resources according to needs
D. Increase human resources training
6. In primary prevention the emphasis of health education should be on:
A. Recognition of health habits and customs
B. Case finding
C. Rehabilitation
D. Early diagnosis
7. The level of prevention directed towards preventing progression of the disease in community is:
A. Primary prevention
B. Secondary prevention
C. Tertiary prevention
D. Primordial prevention
8. The best route to block to prevent disease transmission is:
A. Portal of exit
B. Portal of entry
C. Migration in the body host
D. Vector control
9. Control measure/s directed against the vector:
A. Biological control
B. Mechanical control
C. Chemical control
D. All of the above
10. Some of the advantages of cohort:
A. Establish timing and directionality of events
B. Ideal to study rare exposure
C. Ability to evaluate several clinical outcomes
D. Ease in identifying comparison subjects
11. The DOH has ordered closure/cessation of all commercial blood banking facilities. Blood requirements of patients can be had by:
A. Blood substitutes
B. Voluntary blood donation
C. Component blood production
D. Hospital blood banks
12. In public health, the rate that is used measure illness is known as:
A. Morbidity rate
B. Natality rate
C. Mortality rate
D. Infectivity
13. The rate that reflects socio-economic condition of the community is:
A. Fertility rate
B. Infant mortality
C. Maternal mortality rate
D. A and B
14. Effective preventive medicine means stopping the morbid process when the:
A. Disease is progressing
B. Signs and symptoms are manifest
C. Patient is convalescing
D. Disease has not yet set in

1
15. The primary level of prevention includes:
A. Early case finding
B. Immunization
C. Disability limitation
D. Rehabilitation
16. The first level of preventions is most applicable:
A. Before host and agent interaction occur
B. During the initial interaction between the agent, the host and environment
C. During the convalescent stage of the disease
D. Al of the above
17. Which is the correct statement about Maternal Mortality
A. Higher among the very old and the very young
B. Higher among the primis than secundis
C. Higher among the secundis than the multipara
D. Lower in rural areas
18. Settling tank is a type of:
A. Preliminary treatment of water
B. Primary treatment of water
C. Tertiary treatment of water
D. Secondary treatment of water
19. Water is contaminated with fecal material when the test shows the presence of:
A. Planktons
B. Algae
C. Virus
D. Eschirichia coli
20. It is the most important single test to determine safety of water:
A. Bacteriological exam
B. Physical exam
C. Chemical e exam
D. Biological exam
21. The correct sequence in the process of water purification:
A. Coagulation, sedimentation, filtration distribution
B. Coagulation, sedimentation, filtration, disinfection
C. Sedimentation, coagulation, filtration distribution
D. Filtration, sedimentation, flocculation, disinfection
22. The principal carcinogens related to smoking:
A. Hydrocarbons
B. Nicotine
C. Carbon dioxide
D. Tar
23. Proportion of clinical cases resulting in severe clinical manifestations:
A. Pathogenecity
B. Virulence
C. Immunogenecity
D. Infectivity
24. Ability to produce clinically apparent illness:
A. Pathogenecity
B. Virulence
C. Immunogenecity
D. Infectivity
25. The most useful indicator of long-term nutritional deficiency:
A. Height for age
B. Middle-upper arm circumference
C. Weight for height
D. Weight for age
26. The most useful index for energy nutrient malnutrition in field Dudies relies on:
A. Questionnaire on dietary recall
B. Clinical signs of abnormal bed function
C. Anthropometrics measurement
D. Biochemical test
27. Laws and regulations on food sanitation are implemented to protect the public against fraud and deceit. Some of the restraints are the following EXCEPT:
A. Misbranding
B. Sale of damaged food
C. Food adulteration
D. Adequate food storage
28. The Code of Sanitation of the Philippines is referred to as:
A. PD 1519
B. RA 3573
C. PD 965
D. PD 856
29. The distance of the burial ground from the source of water should be:
A. 100 meters
B. 75 meters
C. 30 feet
D. 10 feet

2
30. The period between the receipt of infection by a host and maximal communicability:
A. Herd immunity
B. Generation time
C. Lead time bias
D. Latency
31. STI has increased in an institute for the last 4 weeks. As public health officer, you should advice:
A. Better sanitation
B. Vaccination
C. Surveillance
D. Chemoprophylaxis
32. A 24 year old male presents with a 4-day history of burning sensation on urination. He also developed profuse creamy urethral discharge. What would
most likely be seen on the gram stain?
A. Numerous WBCs with intracellular gram-negative diplococci
B. Numerous WBCs with intracellular gram-positive diplococci
C. Numerous WBCs
D. Numerous WBCs with no bacteria
33. Resistance of a group to invasion and spread of an infectious agent, based on the immunity of a high proportion of individual members of the group:
A. Protective factors
B. Active immunization
C. Passive Immunization
D. Herd immunity
34. In view of the cardiovascular disease, the principal strategy for their control should be:
A. Health education
B. Environmental control
C. Early diagnosis and treatment
D. Intensive therapy of cases
35. The minimum distance of a pit privy from a well in sandy soil is:
A. 10 meters
B. 15 meters
C. 20 meters
D. 25 meters
36. An essential health services that is available, accessible, affordable and culturally acceptable by the community:
A. Social Care
B. Preventive medicine
C. Medical care
D. Primary health care
37. The essential elements of primary health care:
A. Health education
B. Maternal and child birth
C. Control communicable diseases
D. All of these
38. Separation for the period of communicability
A. Quarantine
B. Isolation
C. Segregation
D. Personal surveillance
39. Which of the following has the greatest impact on community health?
A. Improvement of the standard of living
B. Provision of broad preventive medical measures
C. Provision of specific preventive measures
D. Provision of hospital care
40. The following activities and program supportive of CARI:
A. Nutrition
B. Anti-smoking and anti-pollution
C. Measles immunization
D. All of the above
41. Maternal and Child Health is concern with the health and well being of:
A. Child bearing women
B. Children
C. Potential parents
D. All of the above
42. Limitation of freedom of movement of such well persons
A. Quarantine
B. Isolation
C. Segregation
D. Personal surveillance
43. Requisites for successful parasitism include the following, EXCEPT:
A. Agent
B. Susceptible host
C. Proper climate
D. Satisfactory reservoir
44. The following are forms of quarantine EXCEPT:
A. Segregation
B. Modified quarantine
C. Segregation
D. No exceptions

3
45. The ultimate goal of malaria control eradication is:
A. Complete eradication of the plasmodium parasite
B. Total eradication of the mosquito vector
C. Treatment of all known cases
D. Vigilance against re-introduction
46. Immune globulin is given as preventive measure for many infectious diseases when:
A. Symptoms of the disease first appear
B. Exposure to a frank case has occurred
C. Recovery is taking place
D. Routine immunization is being instituted as well in baby clinics
47. In the Expanded Program of Immunization, BCG is given to:
A. Whenever the health of the child permits
B. Grade I children
C. Newborns
D. children 3-14 months of age
48. The recommended WHO schedule for measles vaccination is at weeks:
A. 6 weeks
B. 10 weeks
C. 3 months
D. 9 months
49. The most important initial management of rabies is:
A. Use of anti-biotic
B. Use of human diploid-cell
C. Irrigate saliva-containing bite wound
D. Catch and kill the dog
50. A 30 year old housemaid complains of progressive abdominal swelling, vague abdominal pain, low fever and weight loss. On PE the abdomen is distended
with a positive shifting dullness, aspiration reveals xanthochromic fluid with a gm/m PROTEIN (blood protein is 5). This suggests:
A. Liver malignancy
B. TB peritonitis
C. Liver cirrhosis
D. Schistosomiasis
51. The following are different characteristics of hepatitis B from hepatitis A. EXCEPT:
A. Longer incubation period
B. Route of entry is mainly parental
C. Onset is more gradual
D. Occurs more in the younger age group
52. The planning of health education activity starts with:
A. Social preparation
B. Hypothesis formulation
C. Needs analysis
D. Budget setting
53. The primary objective of health education is to:
A. Transfer technology to lay personnel
B. Promote health in general
C. Impart knowledge
D. Improve health practices
54. The following is NOT included in the EPI Program of DOH:
A. Chickenpox
B. Measles
C. Diphtheria
D. Pertussis
55. The priority or target group in the anti-TB Program is:
A. Sputum negative but with suggestive of TB
B. Sputum negative cases but with possible x-ray findings
C. Sputum positive cases
D. Sputum negative but with clinical findings suggestive of TB
56. In TB symptomatic patient, how many sputum smears are recommended to establish diagnosis of TB?
A. Two
B. Four
C. Three
D. One
57. The DOH program tutok gamutan refers to:
A. Anti-tetanus vaccination
B. Polio vaccination
C. Micronutrient administration
D. Directly observed treatment strategy for TB
58. The following are forms to reduce susceptibility EXCEPT
A. Health promotion
B. Personal hygiene
C. Treatment
D. Prophylaxis
59. In measles, the period from exposure to appearance of rash is:
A. 3-6 days
B. 7-10 days
C. 12-14 days
D. 16-22 days

4
60. The vector of dengue fever is:
A. Aedes polcilus
B. Anpheles minimus
C. Aedes aegypti
D. Aedes seriation
61. Prevention of hepatitis can be achieved by:
A. Proper hygiene and sanitation
B. Needle and syringe sterilization
C. Screening of blood donors
D. Surveillance of all post-hepatitis
E. All of the above
62. Control has the following characteristics EXCEPT:
A. Cessation of transmission
B. Reduction of incidence
C. Prioritizes areas where transmission is high
D. Does not require total coverage of area
63. The first contact of the community to the health claim as defined by the PHC system is the:
A. Hospital personnel
B. Village health workers
C. Intermediate level health workers
D. Barangay captain
64. The following are true for volunteer community health workers, EXCEPT:
A. Establish linkage between government and non-government organizations
B. Are residents of the community
C. Provide only curative care
D. All of the above
65. The prime yardstick of child health level in the community is:
A. Infant and child mortality rate
B. Number of live births
C. Number of pre-school children
D. Number of health centers
66. Due to occurrence of flood and unsafe water supply, the most useful services that the local health unit can offer is/are:
A. Environmental sanitation
B. Communicable disease control
C. Health education
D. All of the above
67. In investigating an epidemic, cases should be categorized according to:
A. time, place and person
B. agent, host and environment
C. agent, host and date of onset
D. time, person and date of onset
68. At the beginning of the study, the participants are disease free individuals:
A. Cross-sectional
B. Cohort
C. Case-control using prevalent cases
D. Experimental study design
69. The strongest form of medical research study design is:
A. Clinical trial
B. Case report
C. Case-control study design
D. Cohort study design
70. One of the following is not an Analytical Study Design:
A. Prospective Cohort Study Design
B. Retrospective Cohort Study Design
C. Case Control Study Design
D. Population-Based Survey
71. Which of the following study designs may be prematurely terminated if interim analysis are found conclusive:
A. Case-control study design
B. Randomized Controlled Trial
C. Retrospective Cohort Study Design
D. Cross-Sectional Study Design
72. Study design in which exposure to a factor is known to all the study participants:
A. Cross-sectional
B. Cohort
C. Case-control using prevalent cases
D. Experimental study design
73. A study design used to provide prevalence of disease or other health outcomes in certain populations:
A. CASE report
B. Case series
C. Retrospective Cohort Study Design
D. Cross-sectional study design
74. One can be reasonably sure that the hypothesized cause preceded the occurrence of the disease and that disease status did not differentially influence the
selection of subjects by study factor level.
A. Cross-sectional
B. Cohort
C. Case-control using prevalent cases

5
D. Experimental study design
75. The preferred measure to determine causal association:
A. Ratio
B. Difference
C. Prevalence
D. None of the above
76. The measure that reflect the absolute number of cases attributable to the exposure:
A. Ratio
B. Difference
C. Prevalence
D. None of the above
77. The following decreases the prevalence of disease, except:
A. Out migration of susceptible cases
B. Improved immunization against the disease (if the disease is immunizable)
C. Treatment of the disease
D. None of the above
78. The following increases the prevalence of disease:
A. The disease has long duration but the incidence is low
B. The incidence is high but the duration of the disease is very short
C. The duration of the disease is long and the incidence is high
D. None of the above
79. This distribution is widely used because of its capacity to approximate other probabilities.
A. Binomial probability distribution
B. Poisson probability distribution
C. Normal probability distribution
D. None of the above
80. The group that we wish to study
A. Reference population
B. Target population
C. Study population
D. All of the above
81. Definition of mean
A. Summation of all values of sample points divided by the sample size
B. May have exactly the same sample points on both sides of the mean
C. A measure of central location
D. All of the above
E. None of the above
82. The difference between the highest observed value and the lowest observed value
A. Standard deviation
B. Mean
C. Range
D. Median
E. None of the above
83. The definition of variance
A. A measure of how the individual data points behave around the mean.
B. The square of a standard deviation
C. A measure of spread
D. Both b and c is correct
E. All of the above
84. This is an ordered display of each value in a data set together with its frequency, that is, the number of times that value occurs in the data set.
A. Mean
B. Statistical table
C. Proportion
D. None of the above
85. What graphic methods that can be used if a variable is characterized by a numerical attribute, such as systolic blood pressure or birth weight?
A. Horizontal bar diagram
B. Pie chart
C. Histogram
D. Vertical bar diagram
86. The following statement/s are true of bar diagram
A. For comparison of frequencies, whether absolute or relative
B. The magnitude for comparison is represented as bars whose lengths are proportionate to the values.
C. This is used of qualitative variables
D. All of the above
87. Which is true of the histogram
A. This is used for a qualitative variable
B. This is used for a quantitative discrete variable
C. This has the same use as the component bar
D. This is used for a quantitative continuous variable.
E. None of the above
88. The line diagram can be applied to this situation:
A. Qualitative variable
B. Time series
C. Discontinuous variable
D. All of the above
E. None of the above

6
89. Use/s of a scatterplot
A. Correlation data for two quantitative variable
B. Discontinuous variable
C. This is used for qualitative data
D. Both A and C are correct
90. What graphical presentation/s is/are best suited for distributions having a discrete basis of classification?
A. Pie diagram
B. Horizontal bar diagram
C. Vertical bar diagram
D. Both B and C are correct
91. What is a frequency polygon?
A. Used for qualitative data
B. Used for discontinuous data
C. Used for trend data
D. All of the above
E. None of the above
92. Which of the following is/are true of the principles of tabulation.
A. The tables should be simple. Two or three tables is better than one table with many variables
B. Each row and column should be labeled properly with specific units or measures for the data
C. The title should answer the questions what? Where? And when?
D. Totals should be shown
E. All of the above
93. The following descriptive statistics are very sensitive to extreme values.
A. Mean
B. Median
C. Standard deviation
D. Coefficient of variation
E. None of the above
94. The rationale of this measure is to ensure an equal number of sample points on both sides of the central location.
A. Mean
B. Median
C. Coefficient of variation
D. All of the above
E. None of the above
95. The principal strength of the sample is that it is insensitive to very large or very small values
A. Mean
B. Median
C. Mode
D. Coefficient of variation
E. None of the above
96. For distribution of infants according to the number of illness episodes experienced during the calendar year, this presentation is appropriate.
A. Histogram
B. Frequency polygon
C. Vertical bar diagram
D. All of the above
E. None of the above
97. What presentation/s is/are appropriate for the distribution of children enrolled in the feeding program according to sex?
A. Frequency polygon
B. Pie chart
C. Vertical bar diagram
D. All of the above
98. The definition of variance
A. A measure of how the individual data points behave around the mean.
B. The square of a standard deviation
C. A measure of spread
D. All of the above
99. Large data sets are best describe using this/these:
A. Measure of spread
B. Measure of central tendency
C. Tabular presentation
D. Both a and b
E. All of the above
100. When one is interested to compare the variability of the height and the weight of the third year medical student, the best statistics to use is/are:
A. Standard deviation
B. Variance
C. Coefficient of variation
D. All of the above
E. None of the above

***END***

7
Preventive Medicine Answer Key

60. D MPL 50
1. C MPL 90 61. A MPL 75
2. C MPL 90 62. B MPL 75
3. C MPL 75 63. D MPL 75
4. B MPL 75 64. D MPL 90
5. B MPL 90 65. B MPL 90
6. A MPL 75 66. B MPL 75
7. B MPL 50 67. B MPL 75
8. A MPL 75 68. D MPL 90
9. B MPL 75 69. A MPL 50
10. D MPL 75 70. D MPL 75
11. C MPL 90 71. A MPL 75
12. C MPL 75 72. B MPL 75
13. B MPL 75 73. B MPL 75
14. D MPL 75 74. B MPL 75
15. D MPL 75 75. C MPL 75
16. C MPL 50 76. D MPL 90
17. D MPL 50 77. B MPL 75
18. B MPL 50 78. A MPL 75
19. B MPL 75 79. C MPL 75
20. A MPL 90 80. B MPL 75
21. A MPL 75 81. A MPL 90
22. D MPL 75 82. D MPL 90
23. A MPL 75 83. B MPL 90
24. C MPL 75 84. B MPL 90
25. B MPL 75 85. D MPL 90
26. D MPL 75 86. C MPL 75
27. D MPL 75 87. C MPL 90
28. C MPL 75 88. C MPL 75
29. C MPL 75 89. A MPL 75
30. C MPL 75 90. D MPL 90
31. C MPL 50 91. B MPL 75
32. B MPL 50 92. B MPL 90
33. A MPL 75 93. D MPL 90
34. C MPL 75 94. C MPL 90
35. B MPL 75 95. A MPL 90
36. B MPL 50 96. B MPL 90
37. C MPL 75 97. C MPL 75
38. D MPL 75 98. C MPL 75
39. B MPL 50 99. D MPL 75
40. D MPL 50 100.B MPL 90
41. C MPL 75
42. B MPL 50 References: Public Health and
43. B MPL 90 Preventive Medicine 14th ed.; Robert
44. D MPL 90 Melbourne Publisher: Application &
45. C MPL 90 Lange
46. A MPL 90
47. C MPL 75
48. A MPL 75
49. A MPL 75
50. B MPL 75
51. A MPL 90
52. C MPL 75
53. A MPL 50
54. B MPL 50
55. A MPL 75
56. B MPL 75
57. C MPL 75
58. A MPL 75
59. C MPL 50
PREVENTIVE MEDICINE AND COMMUNITY HEALTH

1. The purpose is to limit the incidence of disease by controlling causes and risk factors
A. Primordial prevention
B. Primary prevention
C. Secondary prevention
D. Tertiary prevention

Answer: B

2. The property of a test to identify the proportion of truly ill persons in a population who are identified as ill by a
screening test
A. Sensitivity
B. Specificity
C. Positive predictive value
D. Negative predictive value

Answer : A

3. The probability of a persons having the disease when the test is positive
A. Sensitivity
B. Specificity
C. Positive predictive value
D. Negative predictive value

Answer: C

4. The extent to which a test is measuring what it is intended to measure


A. Reliability
B. Validity
C. Sensitivity
D. Specificity

Answer: B

5. A study that measures the number of persons with influenza in a calendar year
A. Cohort study
B. Case control
C. Cross sectional
D. Case report

Answer: C

6. Stage by which the presence of factors favor the occurrence of disease


A. Stage of susceptibility
B. Stage of presymptomatic disease
C. Stage of clinical disease
D. Stage of disability

Answer: A

7. Modes of horizontal transmission of disease, except


A. Contact
B. Vector
C. Common Vehicle
D. Genetic

Answer:D

8. An infected person is less likely to encounter a susceptible person when a large proportion of the members of the
group are immune
A. Active immunity
B. Passive immunity
C. Herd immunity
D. Specific immunity

Answer: C

1
9. Occurrence in the community of a number of cases of disease that is unusually large or unexpected
A. Endemic
B. Epidemic
C. Pandemic
D. Infection

Answer: B

10. Measures of central tendency, except


A. Mean
B. Median
C. Mode
D. Variance

Answer: D

11. Range of values surrounding the estimate which has a specified probability of including the true population values
A. Standard deviation
B. Standard error
C. Confidence interval
D. Correlation coefficient

Answer: C

12. The probability of rejecting the null hypothesis when it is true


A. Type 1 error
B. Type 2 error
C. Power of a statistical test
D. Level of significance

Answer: A

13. The following are measures of disease frequency, except


A. Incidence rate
B. Prevalence
C. Cumulative incidence
D. Relative risk

Answer: D

14. The proportion of cases of a specified disease or condition which are fatal within a specified time
A. Morbidity rate
B. Case fatality rate
C. Proportionate mortality
D. Death rate

Answer: B

15. The relation between exposure and disease is considered to be causal or etiological in the following, except
A. Dose response relation
B. Cessation of exposure
C. Temporal relation
D. No confounding

Answer: D

16. A study that measures the incidence of a disease


A. Case report
B. Cross sectional
C. Case control
D. Cohort

Answer:D

17. A study wherein bias is less likely to occur


A. Case report
B. Cross sectional
C. Case control

2
D. Cohort

Answer: D

18. The proportion of disease incidence that can be attributed to a specific exposure
A. Relative risk
B. Odds ratio
C. Attributable risk
D. Potential risk

Answer: C

19. All of the following are potential benefits of a randomized clinical trial, except
A. The likelihood that the study groups will be comparable is increased
B. Self-selection for a particular treatment is eliminated
C. External validity of the study is increased
D. Assignment of the next subject cannot be predicted

Answer: C

20. Recall is an example of what type of bias


A. Selection bias
B. Information bias
C. Confounding
D. Systematic

Answer: B

21. Type of design where both exposure and disease are determined simultaneously for each subject
A. Case study
B. Cross sectional study
C. Case control study
D. Cohort study

Answer: B

22. A study is conducted to determine the proportion of persons in the population with PTB using AFB sputum for
diagnosis
A. Case study
B. Cross sectional study
C. Case control study
D. Cohort study

Answer: B

23. Randomization is the best approach in designing a clinical trial in order to


A. Achieve predictability
B. Achieve unpredictability
C. Achieve blinding
D. Limit confounding

Answer: B

24. Type of sampling whereby subjects are assigned according to a factor that would influence the outcome of a study
A. Simple random sampling
B. Systematic sampling
C. Stratified random sampling
D. Cluster sampling

Answer: C

25. The extent to which a specific health care treatment, service, procedure, program, or other intervention produces a
beneficial result under ideal controlled conditions is its
A. Effectiveness
B. Efficacy
C. Efficiency
D. Effect modification

3
Answer: B

26. What is the desired body weight of a 7 month old infant weighing 3600 gms at birth?
A. 8.8 kgs
B. 9 kgs
C. 6.5 kgs
D. 7.8 ks

Answer: D

27. Infants double their weight at


A. 6-7 mos
B. 9-10 mos
C. 5-6 mos
D. 3-4 mos

Answer: C

28. What is the total energy requirement for a 50 kg housewife without househelp?
A. 2000 cal/day
B. 1800 cal/day Answer: C
C. 2100 cal/day
D. 2200 cal/day

29. The symptom that appears to consistently differentiate between PTB and non-TB respiratory disease

A. Night sweats
B. Anorexia
C. Chronic cough
D. Hemoptysis

Answer: C

30. Asymptomatic PTB is most prevalent in what group?


A. Infants
B. Smokers
C. Young Children
D. Older age group

Answer: D

31. What should be the first test done when confronted with a patient in whom there is clinical suspicion of
PTB?
A. Chest X-ray
B. Sputum AFB
C. Sputum GS/CS
D. All of the above

Answer:

32. What is the ideal timing of the three sputum samples?


A. First specimen after time of consultation at home, Second specimen early morning sputum collected by the patient
and Third spot specimen upon submission of second sputum
B. First specimen at home, Second specimen on the spot at time of consultation and Third spot specimen upon
submission of second sputum.
C. First specimen on the spot at time of consultation, Second specimen early morning sputum collected by the patient
and Third spot specimen upon submission of second sputum.
D. First specimen on the spot at time of consultation, Second specimen two days after consultation collected by the
patient at home and Third spot specimen upon submission of second sputum.

Answer: C

33. A Sputum AFB result of (++) means

4
A. 3-9 bacilli in entire smear D. 1-9 bacilli /10 OIF
B. 1-9 bacilli /OIF
C. 1-2 bacilli in entire smear Answer: D

34. What is a significant minimum microscopy result for the presumptive diagnosis of PTB?
A. A report of AFB ++ or more for any one of the submitted sputum specimens
B. If at least one of the 3 specimens is positive
C. A report of AFB +++ or more for any one of the submitted sputum specimens
D. A report of AFB + or more for any one of the submitted sputum specimens

Answer: A

35. TB cultures should be done in any of the following situations except


A. Smear (+) patients with fall and rise phenomenon
B. All cases of previously treated for >3months but <6months
C. Smear (+) patients with symptoms highly suggestive of PTB and suggestive C-Xrays
D. All cases of relapse

Answer: C

36. A patient with positive PTB exposure and PPD but lacking in signs of active disease and target organ damage is
classified by the American Thoracic Society as
A. PTB I
B. PTB II
C. PTB II
D. PTB IV

Answer: B

37. In the prevention of Cardiovascular disease, salt restriction should be less than how many grams per day?
A. 2 gms
B. 3gms
C. 4gms
D. 5gms

Answer: D

38. In Cardiovascular disease prevention, one should progressively increase moderate physical activity to
A. 30mins once a week
B. 30 mins twice a week
C. 30 mins three times a week
D. 30 mins daily

Answer: D

39. In patients with history of TIA, heart attack or diabetes, the goal in Blood pressure is
A. < 140 and <90
B. <130 and <80
C. <120 and <70
D. < 110 and < 60

Answer: B

40. Leading cause of Diarrheal disease


A. Enterotoxigenic Escherichia coli
B.Salmonella (non-typhoid)
C.Rotavirus
D. Campylobacter jejuni

Answer: C

41. What degree of dehydration is often seen in a child exhibiting more thirst and sunken eyeballs?
A. No signs
B. Some signs of dehydration
C. Severe signs of dehydration
D. Equivocal

Answer: B

5
42. In Treatment Plan A, the amount of ORS to be given to a child less than 2 y.o. after each loose stool is
A. 100-200ml
B. 50-100ml
C. 300ml
D. 250ml

Answer: B

43. If the child vomits, you may continue slowly giving small amounts of ORS solution after a minimum of
A. 1 hr
B. 30 mins
C. 2 hrs
D. 10 mins

Answer: D
Reference: Association of Medical Colleges Foundation, Control of Diarrheal Disease
MPL:0.50

44. How many grams Sodium Chloride is in the standard World Health Organization ORS formula?
A. 5
B. 20
C. 1.9
D. 3.5

Answer: D

45. Which of the following is true regarding appropriate Nutritional Therapy during Diarrhea?
A. decreases stool output
B. shortens duration of illness
C. allows significant weight gain
D. All of the above

Answer: D

46. Judge Reyes wants for all his children and their respective families to live with him and his wife until the
time of his death. This is an example of what structure of family?
A. Nuclear
B. Extended
C. Single Parent
D. Blended

Answer: B

47.Juan and Maria with their five year old son live in their own home and are managing to survive with
Juans daily wage. This is an example of what structure of family
A. Nuclear
B. Extended
C. Single Parent
D. Blended

Answer: A

48. Start of the family life cycle


A. Newly Married Couple
B. Family With Young Children
C. Unattached Young Adult
D. Family With Adolescents

Answer: C

49. Mammography should be done annually in women of what age?


A. 50 y.o. and above
B. 60 y.o. and above
C. 45 y.o. and above
D. 30 y.o. and above
Answer: A

6
50. Women with prior pregnancy affected by neural tube defects who are planning pregnancy should take this
chemo-prophylactic
A. Vit. C
B. Retinoin
C. Vit. B complex
D. Folic Acid
E. Ferrous Sulfate

Answer: D

51. An act penalizing the refusal of hospitals and medical clinics to administer appropriate initial medical
treatment and support in emergency or serious cases, amending for the purpose batas pambansa bilang 702,
otherwise known as "an act prohibiting the demand of deposits or advance payments for the confinement or
treatment of patients in hospitals and medical clinics in certain cases". Is also known as:
A Ra 8347
B. Ra 3742
C. Ra 8344
D. Ra 7844

Answer: C

52. An act to advance corneal transplantation in the Philippines, amending for the purpose republic act
numbered seven thousand one hundred and seventy (R.A. no. 7170), otherwise known as the organ donation
act . This was enacted on:
A. January 14, 1996
B. December 15, 1997
C. August 14, 1998
D. February 20,1995

Answer: D

53.Republic act. 7875 is also known as the :


a. National organ donation act of 1995
b. National health insurance act of 1995
c. Magna carta for health workers
d. None of the above

Answer: B

54.Republic Act 8423


A. created the PITAHC
B. provided for the Traditional and Alternative Health Development Fund
C. provided for the intensive and continuous scientific study on the herbal plants
D. all of the above

Answer: D

55.Consumer act of the Philippines or RA 7394 involves all of the following, except:
A. Protection against hazards to health and safety.
B. Provision of information and education to facilitate sound choice and the proper exercise of rights
by the consumer.
C. Involvement of consumer representatives in the formulation of social and economic policies.
D. None of the above

Answer: D

56.The tool of family assessment known as SCREEM evaluates the following aspects of family life
A. Social
B. Cultural
C. Religious
D. All of the above

Answer: D

57.True about a family genogram


A. Graphically represents at least 3 generations
B. Includes even the deceased family members
C. Completely depicts family functioning

7
D. A and B

Answer: D

58. Family assessment instruments include


A. Family genogram
B. Family APGAR
C. Clinical biography and life events
D. All of the above

Answer: D

59.APGAR family assessment is interpreted by means of


A. Scoring
B. Comparing with a standard table
C. Using a scale of wellness
D. Consultation with a family psychologist

Answer: A

60.The APGAR family assessment measures


A. The newborns color, reflex, and cry
B. Satisfaction of members in the relationships within the family
C. The income of the family
D. The level of communication between husband and wife

Answer: B

61.True about family assessment


A. Should be done for all conscious patients and omitted for comatose patients
B. All the tools of assessment must be used
C. An interview with one member of the family is adequate
D. None of the above

Answer: D

62.Normal family function includes


A. Provide support to each other
B. Create rules that govern the conduct of family
C. Adapting to changes in the environment
D. All of the above

Answer: D

63.The statement that a family is a system means that


A. members have inherent inter-dependence
B. parents take the lead role
C. changes affect each member
D. A and C

Answer: D

64.The family as a unit of care is very special because


A. there is lifelong involvement
B. there are shared genetic and developmental attributes
C. they are confined to one household
D. A and B

Answer: D

65.Members of the nuclear family


A. parents
B. dependent children
C. spouses of the dependent children
D. A and B

Answer: D

8
66.Data on the family background is not needed when
A. Patient is comatose
B. Patient is caucasian
C. Patient is anti-social
D. None of the above

Answer: D

67. Basic areas of family function


A. Biologic
B. Economic
C. Educational
D. All of the above

Answer: D

68.It is the responsibility of the attending physician to explain the following


A. etiology of illness
B. pathophysiology of the illness
C. trajectory of outcome of the illness
D. All of the above

Answer: D

69.The illness trajectory begins with


A. the confirmatory laboratory results
B. the onset of symptoms
C. the onset of therapeutic efforts
D. recovery phase

Answer: B

70.Critical issues in choosing a therapeutic plan


A. financial cost
B. psychological state and preparedness of the patient and family
C. lifestyle and cultural characteristics of a family
D. All of the above

Answer: D

71.True about impact of acute illness in the family


A. emotions are high, and can lead to anger
B. family has no time for psychological adjustment
C. family members become sick in the course of the crisis
D. A and B

Answer: D

72.True about the illness trajectory


A. it is the pathological process of coping with illness
B. knowledge of the trajectory allows the physician to predict and anticipate family response to
illness
C. it is the normal course of the psychosocial aspect of disease
D. B and C

Answer: D

73. The following statements are related to the impact of illness


A. Severe illness in parents place children of a family at great psychosocial difficulty
B. Illness sets in motion the processes that are disruptive of family life
C. Illness that is prolonged results in changes in family structure, roles and functions
D. All of the above

Answer: D

74.The five stages of family illness trajectory are the following except
A. Recovery phase
B. Onset of Illness to diagnosis

9
C. Termination phase
D. Major therapeutic efforts

Answer: C

75. Investigating the illness means


A. exploring the meaning of illness to patient and family
B. obtaining clinical history
C. obtaining laboratory test results
D. All of the above

Answer: A

76. The illness belief model may be influenced by


A. scientific medicine
B. religious beliefs
C. popular account
D. All of the above

Answer: D

77. During the impact phase, the physicians responsibility include


A. Making clinical judgment about the amount of information the patient can absorb
B. Giving support and continuity of care
C. Clarify etiology of illness to address any feeling of guilt among family members
D. All of the above

Answer: D

78. Philhealth is
B. is a government collecting agency
C. is a government agency mandated by law to implement the NHIP
D. All of the above

Answer: C

79. Universal coverage means that PhilHealth aims


to give medical coverage
A. to all Filipino citizens
B. to all Filipino citizens and foreign nationals
C. to natural born filipinos only
D. all of the above

Answer: A

80. Participation of health care providers in the NHIP


A. is compulsory
B. is a privilege
C. both A & B
D. neither A nor B

Answer: B

81. Surgical procedures covered by philhealth


A.laparoscopic procedures
B.caesarian section
C.excision biopsy
D.All of the above

Answer: D

82. Current outpatient benefits include


A. chemotherapy, hemodialysis, minor surgical procedures
B. chemotherapy, laboratory services,radiotherapy
C. hemodialysis, dental extraction, cataract extraction
D. all of the above

Answer: A

10
83. This law refers to the compulsory health
insurance of the government
A. Medicare
B. Republic Act 7872
C. National Health Insurance Act of 1995
D. None of the above
Answer: C

84. NHIP ensures that health services are


A. affordable,acceptable,available,accessible
B. affordable,adjustable,admirable,negotiable
C. affordable and world class
D. none of the above

Answer: A

85 PhilHealth sets standards, guidelines and


procedures prior to allowing doctors and hospitals
to become part of the NHIP. This process is called
A. evaluation
B. accreditation
C. recognition
D. acceleration

Answer: B

86. PhilHealth gives medical coverage to dependents of


members. Dependents are the members
A. parents over 60, children below 21
B. parents over 65, children below 21
C. parents over 60, children below 18
D. parents over 65, children of any age

Answer: A

87. Health care provider in the definition of NHIP


can be any of the following
A. doctor, under-board nurse, midwife
B. hospital, CBHO, HMO
C. doctor, hospital, faith healer
D. surgeon, anesthesiologist, medical intern

Answer: B

88. Non-paying philhealth member refers to


A. a member who has reached age 60
B. retiree who has paid 120 months of continuous
contributions
C. delinquent members
D. A & B

Answer: D

89. Surgical procedures NOT covered by philhealth


A. laparoscopic procedures
B. caesarian section
C. silicone implantation, breast
D. excision biopsy

Answer: C

90. In the current EPI, a fully immunized child has


A. 1 BCG, 3 DPT, 3 OPV,1 MMR, 3 Hep B vaccine before 12 months of age
B. 2 BCG, 1 Measles vaccine, 3 DPT, 3 OPV before 12 months of age
C. 1 BCG, 3 DPT, 3 OPV, 1 Measles before 1 year of age
D. 1 BCG, 3 DPT, 3 OPV, 1 Measles, 3 Hep B vaccine before 1 year of age

11
Answer: D

91. The following are the target population of the EPI


A. all poor children at high risk areas
B. children 1-5 years old in measles high risk areas
C. all children without vaccination
D. infants 0 12 mos old, born to un-immunized mother

Answer: B

92. EPI was successful because it utilized different


implementation strategies which include
A. monitoring, surveillance, health education
B. fee collection, social marketing
C. social mobilization, routine immunization day
D. A & C

Answer: D

93. Outbreak response means that


A. giving polio vaccine to children who have not
completed their OPV once a case of polio is reported
B. giving OPV to all children under 5 regardless
of immunization status once a case of polio is reported
C. giving measles immunization to contacts of a
child with measles regardless of immunization status
D.giving all the EPI vaccinations to a child once
a case of polio is reported in the area

Answer: B
MPL:0.60

94. In all levels of the government organization,


there are four distinct personnel in the EPI,
they are:
A. immunization officer, health educator
B. cold chain manager, disease surveillance officer
C. marketing officer, purchasing officer
D. A & B
Answer: D

95. It is given to women of child-bearing age


A. Tetanus immunoglobulin
B. Tetanus toxoid
C. Measles vaccine
D. BCG booster

Answer: B

96. When the child has the following condition,


immunization is absolutely contraindicated
A. HIV infection
B. Adverse reaction to the previous immunization
C. A & B
D. None of the above

Answer: C

97.The goal(s) of the EPI is/are


A. Government revenue
B. Healthier workforce
C. Disease prevention
D. B & C

Answer: C

12
98. You may postpone immunization when the following
conditions are present
A. when the child is hospitalized
B. when the child is febrile
C. when the child is taking antibiotics
D. A & B

Answer: D

99. You may proceed with immunization even when


A. the child has low grade fever
B. the child is preterm
C. then child is malnourished
D. All of the above

Answer: D

100. The following are examples of active immunization


A. BCG & OPV
B. BCG & DPT
C. BCG & Hepa B
D. All of the above

Answer: D

1. The main thrust in the control of diarrhea cases by the Department of Health is:
A. Environmental Sanitation
B. Oral Rehydration Therapy with ORESOL
C. Public education
D. Immunization

2. Which of the following child survival strategies complement Oral Rehydration by decreasing the incidence
and severity of diarrheal infection?
A. Growth Monitoring
B. Immunization
C. Breastfeeding
D. Mothers education on child care

3. The probability that a positive result is truly indicative of the disease or condition is:
A. None of these
B. Sensitivity
C. Specificity
D. positive

4. Public health services must give emphasis on:


A. Prevention, promotion and maintenance of health
B. Treatment of diseases
C. Rehabilitation of the differently abled individuals
D. Provision of emergency and first aid services

5. In public health, the most frequent problem is scarcity of resources. This can be best met by:
A. Transfer of funds
B. Compete for national funds
C. Allocate resources according to needs
D. Increase human resources training

6. In primary prevention the emphasis of health education should be on:


A. Recognition of health habits and customs
B. Case finding
C. Rehabilitation
D. Early diagnosis

7. The level of prevention directed towards preventing progression of the disease in community is:
A. Primary prevention
B. Secondary prevention

13
C. Tertiary prevention
D. Primordial prevention

8. The best route to block to prevent disease transmission is:


A. Portal of exit
B. Portal of entry
C. Migration in the body host
D. Vector control

9. Control measure/s directed against the vector:


A. Biological control
B. Mechanical control
C. Chemical control
D. All of the above

10. Some of the advantages of cohort:


A. Establish timing and directionality of events
B. Ideal to study rare exposure
C. Ability to evaluate several clinical outcomes
D. Ease in identifying comparison subjects

11. The DOH has ordered closure/cessation of all commercial blood banking facilities. Blood requirements of
patients can be had by:
A. Blood substitutes
B. Voluntary blood donation
C. Component blood production
D. Hospital blood banks

12. In public health, the rate that is used measure illness is known as:
A. Morbidity rate
B. Natality rate
C. Mortality rate
D. Infectivity

13. The rate that reflects socio-economic condition of the community is:
A. Fertility rate
B. Infant mortality
C. Maternal mortality rate
D. A and B

14. Effective preventive medicine means stopping the morbid process when the:
A. Disease is progressing
B. Signs and symptoms are manifest
C. Patient is convalescing
D. Disease has not yet set in

15. The primary level of prevention includes:


A. Early case finding
B. Immunization
C. Disability limitation
D. Rehabilitation

16. The first level of preventions is most applicable:


A. Before host and agent interaction occur
B. During the initial interaction between the agent, the host and environment
C. During the convalescent stage of the disease
D. Al of the above

17. Which is the correct statement about Maternal Mortality


A. Higher among the very old and the very young
B. Higher among the primis than secundis
C. Higher among the secundis than the multipara
D. Lower in rural areas

18. Settling tank is a type of:


A. Preliminary treatment of water

14
B. Primary treatment of water
C. Tertiary treatment of water
D. Secondary treatment of water

19. Water is contaminated with fecal material when the test shows the presence of:
A. Planktons
B. Algae
C. Virus
D. Eschirichia coli

20. It is the most important single test to determine safety of water:


A. Bacteriological exam
B. Physical exam
C. Chemical e exam
D. Biological exam

21. The correct sequence in the process of water purification:


A. Coagulation, sedimentation, filtration distribution
B. Coagulation, sedimentation, filtration, disinfection
C. Sedimentation, coagulation, filtration distribution
D. Filtration, sedimentation, flocculation, disinfection

22. The principal carcinogens related to smoking:


A. Hydrocarbons
B. Nicotine
C. Carbon dioxide
D. Tar

23. Proportion of clinical cases resulting in severe clinical manifestations:


A. Pathogenecity
B. Virulence
C. Immunogenecity
D. Infectivity

24. Ability to produce clinically apparent illness:


A. Pathogenecity
B. Virulence
C. Immunogenecity
D. Infectivity

25. The most useful indicator of long-term nutritional deficiency:


A. Height for age
B. Middle-upper arm circumference
C. Weight for height
D. Weight for age

26. The most useful index for energy nutrient malnutrition in field Dudies relies on:
A. Questionnaire on dietary recall
B. Clinical signs of abnormal bed function
C. Anthropometrics measurement
D. Biochemical test

27. Laws and regulations on food sanitation are implemented to protect the public against fraud and deceit.
Some of the restraints are the following EXCEPT:
A. Misbranding
B. Sale of damaged food
C. Food adulteration
D. Adequate food storage

28. The Code of Sanitation of the Philippines is referred to as:


A. PD 1519
B. RA 3573
C. PD 965
D. PD 856

29. The distance of the burial ground from the source of water should be:
A. 100 meters
B. 75 meters

15
C. 30 feet
D. 10 feet

30. The period between the receipt of infection by a host and maximal communicability:
A. Herd immunity
B. Generation time
C. Lead time bias
D. Latency

31. STI has increased in an institute for the last 4 weeks. As public health officer, you should advice:
A. Better sanitation
B. Vaccination
C. Surveillance
D. Chemoprophylaxis

32. A 24 year old male presents with a 4-day history of burning sensation on urination. He also developed
profuse creamy urethral discharge. What would most likely be seen on the gram stain?
A. Numerous WBCs with intracellular gram-negative diplococci
B. Numerous WBCs with intracellular gram-positive diplococci
C. Numerous WBCs
D. Numerous WBCs with no bacteria

33. Resistance of a group to invasion and spread of an infectious agent, based on the immunity of a high
proportion of individual members of the group:
A. Protective factors
B. Active immunization
C. Passive Immunization
D. Herd immunity

34. In view of the cardiovascular disease, the principal strategy for their control should be:
A. Health education
B. Environmental control
C. Early diagnosis and treatment
D. Intensive therapy of cases

35. The minimum distance of a pit privy from a well in sandy soil is:
A. 10 meters
B. 15 meters
C. 20 meters
D. 25 meters

36. An essential health services that is available, accessible, affordable and culturally acceptable by the
community:
A. Social Care
B. Preventive medicine
C. Medical care
D. Primary health care

37. The essential elements of primary health care:


A. Health education
B. Maternal and child birth
C. Control communicable diseases
D. All of these

38. Separation for the period of communicability


A. Quarantine
B. Isolation
C. Segregation
D. Personal surveillance

39. Which of the following has the greatest impact on community health?
A. Improvement of the standard of living
B. Provision of broad preventive medical measures
C. Provision of specific preventive measures
D. Provision of hospital care

16
40. The following activities and program supportive of CARI:
A. Nutrition
B. Anti-smoking and anti-pollution
C. Measles immunization
D. All of the above

41. Maternal and Child Health is concern with the health and well being of:
A. Child bearing women
B. Children
C. Potential parents
D. All of the above

42. Limitation of freedom of movement of such well persons


A. Quarantine
B. Isolation
C. Segregation
D. Personal surveillance

43. Requisites for successful parasitism include the following, EXCEPT:


A. Agent
B. Susceptible host
C. Proper climate
D. Satisfactory reservoir

44. The following are forms of quarantine EXCEPT:


A. Segregation
B. Modified quarantine
C. Segregation
D. No exceptions

45. The ultimate goal of malaria control eradication is:


A. Complete eradication of the plasmodium parasite
B. Total eradication of the mosquito vector
C. Treatment of all known cases
D. Vigilance against re-introduction

46. Immune globulin is given as preventive measure for many infectious diseases when:
A. Symptoms of the disease first appear
B. Exposure to a frank case has occurred
C. Recovery is taking place
D. Routine immunization is being instituted as well in baby clinics

47. In the Expanded Program of Immunization, BCG is given to:


A. Whenever the health of the child permits
B. Grade I children
C. Newborns
D. children 3-14 months of age

48. The recommended WHO schedule for measles vaccination is at weeks:


A. 6 weeks
B. 10 weeks
C. 3 months
D. 9 months

49. The most important initial management of rabies is:


A. Use of anti-biotic
B. Use of human diploid-cell
C. Irrigate saliva-containing bite wound
D. Catch and kill the dog

50. A 30 year old housemaid complains of progressive abdominal swelling, vague abdominal pain, low fever
and weight loss. On PE the abdomen is distended with a positive shifting dullness, aspiration reveals
xanthochromic fluid with a gm/m PROTEIN (blood protein is 5). This suggests:
A. Liver malignancy
B. TB peritonitis
C. Liver cirrhosis
D. Schistosomiasis

17
51. The following are different characteristics of hepatitis B from hepatitis A. EXCEPT:
A. Longer incubation period
B. Route of entry is mainly parental
C. Onset is more gradual
D. Occurs more in the younger age group

52. The planning of health education activity starts with:


A. Social preparation
B. Hypothesis formulation
C. Needs analysis
D. Budget setting

53. The primary objective of health education is to:


A. Transfer technology to lay personnel
B. Promote health in general
C. Impart knowledge
D. Improve health practices

54. The following is NOT included in the EPI Program of DOH:


A. Chickenpox
B. Measles
C. Diphtheria
D. Pertussis

55. The priority or target group in the anti-TB Program is:


A. Sputum negative but with suggestive of TB
B. Sputum negative cases but with possible x-ray findings
C. Sputum positive cases
D. Sputum negative but with clinical findings suggestive of TB

56. In TB symptomatic patient, how many sputum smears are recommended to establish diagnosis of TB?
A. Two
B. Four
C. Three
D. One

57. The DOH program tutok gamutan refers to:


A. Anti-tetanus vaccination
B. Polio vaccination
C. Micronutrient administration
D. Directly observed treatment strategy for TB

58. The following are forms to reduce susceptibility EXCEPT


A. Health promotion
B. Personal hygiene
C. Treatment
D. Prophylaxis

59. In measles, the period from exposure to appearance of rash is:


A. 3-6 days
B. 7-10 days
C. 12-14 days
D. 16-22 days

60. The vector of dengue fever is:


A. Aedes polcilus
B. Anpheles minimus
C. Aedes aegypti
D. Aedes seriation

61. Prevention of hepatitis can be achieved by:


A. Proper hygiene and sanitation
B. Needle and syringe sterilization
C. Screening of blood donors

18
D. Surveillance of all post-hepatitis
E. All of the above

62. Control has the following characteristics EXCEPT:


A. Cessation of transmission
B. Reduction of incidence
C. Prioritizes areas where transmission is high
D. Does not require total coverage of area

63. The first contact of the community to the health claim as defined by the PHC system is the:
A. Hospital personnel
B. Village health workers
C. Intermediate level health workers
D. Barangay captain

64. The following are true for volunteer community health workers, EXCEPT:
A. Establish linkage between government and non-government organizations
B. Are residents of the community
C. Provide only curative care
D. All of the above

65. The prime yardstick of child health level in the community is:
A. Infant and child mortality rate
B. Number of live births
C. Number of pre-school children
D. Number of health centers

66. Due to occurrence of flood and unsafe water supply, the most useful services that the local health
unit can offer is/are:
A. Environmental sanitation
B. Communicable disease control
C. Health education
D. All of the above

67. In investigating an epidemic, cases should be categorized according to:


A. time, place and person
B. agent, host and environment
C. agent, host and date of onset
D. time, person and date of onset

68. At the beginning of the study, the participants are disease free individuals:
A. Cross-sectional
B. Cohort
C. Case-control using prevalent cases
D. Experimental study design

69. The strongest form of medical research study design is:


A. Clinical trial
B. Case report
C. Case-control study design
D. Cohort study design

70. One of the following is not an Analytical Study Design:


A. Prospective Cohort Study Design
B. Retrospective Cohort Study Design
C. Case Control Study Design
D. Population-Based Survey

71. Which of the following study designs may be prematurely terminated if interim analysis are found
conclusive:
A. Case-control study design
B. Randomized Controlled Trial
C. Retrospective Cohort Study Design
D. Cross-Sectional Study Design

72. Study design in which exposure to a factor is known to all the study participants:
A. Cross-sectional
B. Cohort

19
C. Case-control using prevalent cases
D. Experimental study design

73. A study design used to provide prevalence of disease or other health outcomes in certain populations:
A. CASE report
B. Case series
C. Retrospective Cohort Study Design
D. Cross-sectional study design

74. One can be reasonably sure that the hypothesized cause preceded the occurrence of the disease
and that disease status did not differentially influence the selection of subjects by study factor level.
A. Cross-sectional
B. Cohort
C. Case-control using prevalent cases
D. Experimental study design

75. The preferred measure to determine causal association:


A. Ratio
B. Difference
C. Prevalence
D. None of the above

76. The measure that reflect the absolute number of cases attributable to the exposure:
A. Ratio
B. Difference
C. Prevalence
D. None of the above

77. The following decreases the prevalence of disease, except:


A. Out migration of susceptible cases
B. Improved immunization against the disease (if the disease is immunizable)
C. Treatment of the disease
D. None of the above

78. The following increases the prevalence of disease:


A. The disease has long duration but the incidence is low
B. The incidence is high but the duration of the disease is very short
C. The duration of the disease is long and the incidence is high
D. None of the above

79. This distribution is widely used because of its capacity to approximate other probabilities.
A. Binomial probability distribution
B. Poisson probability distribution
C. Normal probability distribution
D. None of the above

80. The group that we wish to study


A. Reference population
B. Target population
C. Study population
D. All of the above

81. Definition of mean


A. Summation of all values of sample points divided by the sample size
B. May have exactly the same sample points on both sides of the mean
C. A measure of central location
D. All of the above
E. None of the above

82. The difference between the highest observed value and the lowest observed value
A. Standard deviation
B. Mean
C. Range
D. Median
E. None of the above

20
83. The definition of variance
A. A measure of how the individual data points behave around the mean.
B. The square of a standard deviation
C. A measure of spread
D. Both b and c is correct
E. All of the above

84. This is an ordered display of each value in a data set together with its frequency, that is, the number of
times that value occurs in the data set.
A. Mean
B. Statistical table
C. Proportion
D. None of the above

85. What graphic methods that can be used if a variable is characterized by a numerical attribute, such as
systolic blood pressure or birth weight?
A. Horizontal bar diagram
B. Pie chart
C. Histogram
D. Vertical bar diagram

86. The following statement/s are true of bar diagram


A. For comparison of frequencies, whether absolute or relative
B. The magnitude for comparison is represented as bars whose lengths are proportionate to the values.
C. This is used of qualitative variables
D. All of the above

87. Which is true of the histogram


A. This is used for a qualitative variable
B. This is used for a quantitative discrete variable
C. This has the same use as the component bar
D. This is used for a quantitative continuous variable.
E. None of the above

88. The line diagram can be applied to this situation:


A. Qualitative variable
B. Time series
C. Discontinuous variable
D. All of the above
E. None of the above

89. Use/s of a scatterplot


A. Correlation data for two quantitative variable
B. Discontinuous variable
C. This is used for qualitative data
D. Both A and C are correct

90. What graphical presentation/s is/are best suited for distributions having a discrete basis of classification?
A. Pie diagram
B. Horizontal bar diagram
C. Vertical bar diagram
D. Both B and C are correct

91. What is a frequency polygon?


A. Used for qualitative data
B. Used for discontinuous data
C. Used for trend data
D. All of the above
E. None of the above

92. Which of the following is/are true of the principles of tabulation.


A. The tables should be simple. Two or three tables is better than one table with many variables
B. Each row and column should be labeled properly with specific units or measures for the data
C. The title should answer the questions what? Where? And when?
D. Totals should be shown
E. All of the above

21
93. The following descriptive statistics are very sensitive to extreme values.
A. Mean
B. Median
C. Standard deviation
D. Coefficient of variation
E. None of the above

94. The rationale of this measure is to ensure an equal number of sample points on both sides of the central
location.
A. Mean
B. Median
C. Coefficient of variation
D. All of the above
E. None of the above

95. The principal strength of the sample is that it is insensitive to very large or very small values
A. Mean
B. Median
C. Mode
D. Coefficient of variation
E. None of the above

96. For distribution of infants according to the number of illness episodes experienced during the calendar
year, this presentation is appropriate.
A. Histogram
B. Frequency polygon
C. Vertical bar diagram
D. All of the above
E. None of the above

97. What presentation/s is/are appropriate for the distribution of children enrolled in the feeding program
according to sex?
A. Frequency polygon
B. Pie chart
C. Vertical bar diagram
D. All of the above

98. The definition of variance


A. A measure of how the individual data points behave around the mean.
B. The square of a standard deviation
C. A measure of spread
D. All of the above

99. Large data sets are best describe using this/these:


A. Measure of spread
B. Measure of central tendency
C. Tabular presentation
D. Both a and b
E. All of the above

100. When one is interested to compare the variability of the height and the weight of the third year medical
student, the best statistics to use is/are:
A. Standard deviation
B. Variance
C. Coefficient of variation
D. All of the above
E. None of the above

22
BLUEPRINT for PREVENTIVE and SOCIAL MEDICINE
TEST QUESTIONS

RECALL ANALYSIS SYNTHEIS TOTAL


PROBLEM-
SOLVING
I. PRIMARY
PREVENTION
Definitions
Preventive medicine 7 4 11
Public health
Epidemiology
Domestic violence
Functions
Public health 8 12 20
Garbage disposal
Immunization
The Family

II. SECONDARY AND


TERTIARY PREVENTION
Lifestyle diseases 3 5 8
Infectious diseases 10 6 16
Nutrition 3 3
Domestic violence 3 7 10

III. EPIDEMIOLOGY AND 15 12 2 29


STATISTICS

IV. SOCIAL 3 3
ACCOUNTABILITY
TOTAL 52 46 2 100
Preventive Medicine and Community Health: Test Blue Print

Recall Comprehension Analysis/Problem Question No.


Solving
Objectives 1 1
1. To distinguish the
different levels of
disease prevention
2. To Identify the 3 10,11,12
different methods
used in biostatistics
3. To describe the 3 2,3,4
different
measurements and
error of
measurements
4. To identify 5,16,17,21,22,
different study
designs
To discuss the 2 19,23
concepts of study
design selection
5. To explain the 1 6
spectrum of
infection
6. To identify the 1 7
different
mechanisms of
disease transmission
7. To identify the 1 9
different disease
occurrences
8. To identify the 1 8
different type of
immunity
9. To discuss the 2 13,14
different measures
of disease frequency
10. To describe 1 15
disease causation in
the development of
disease
11. To describe the 1 12
different concepts in
hypothesis testing.
12. To identify the 1 24
different sampling
methods
13.. To explain the 1 18
different measures
of incidence in
relation to exposure
14. To describe the 1 20
different elements of
a research process
15. To describe the 1 25
different outcomes
of controlled studies
16. To compute the 1 26,27
desired body weight
based on age
17. To be able to 1 28
compute for Total
energy requirement
based on weight and
activity
18. To identify the 2 29,30
prevalence of PTB
in terms of
population and
symptoms
19. To describe the 5 31,32,33,34,35
basic concepts
behind PTB
diagnosis and
treatment
20. To Identify the 1 36
different
classifications of
PTB
21. To identify the 37,38
different preventive
approaches for
Cardiovascular
disease

22. To describe the


goals of
management of 1 39
CVD
23. To identify the 1 40
leading causes of
Diarrheal disease
24.To enumerate the 5 41,42,43,44,45
basic concepts in the
diagnosis and
management of
diarrheal disease
25. To identify the 3 46,47,65
different family
structures and their
components
26. To describe the 1 48
different stages in
the family life cycle
27. To identify the 1 49,50
different
interventions in a
periodic health exam
based on age and
risk factors
28. To identify the 5 51,52,53,54,55
different republic
acts and their
implementation
date, pertinent to
Community Health
29. To describe the 7 56,57,58,59,60,61.66
different tools and
concepts used in
family assessment
30. To describe 4 62,63,64,67
family function and
systems
31. To describe the 8 68,69,70,72,74,75,76,77
components and
concepts of illness
trajectory
32. To describe the 2 71,73
impact of illness on
the family
33. To describe the 9 78,79,80,81,
basic concepts and 82,85,86,88,89
coverage of
PhilHealth
34. To discuss the 3 83,84,87
implications of the
National Health
Insurance Act
35. To identify the 1 90
component s of
Expanded Program
for Immunization
(EPI)
36. To discuss the 91,92,93,94,95,
basic concepts and 10 96,97,98,99,100
importance of EPI

You might also like